Stresses in Beams (Basic Topics)

You might also like

Download as pdf or txt
Download as pdf or txt
You are on page 1of 108

05Ch05.

qxd 9/24/08 4:59 AM Page 389

5
Stresses in Beams
(Basic Topics)

Longitudinal Strains in Beams d

Problem 5.4-1 Determine the maximum normal strain âmax produced in


a steel wire of diameter d  1/16 in. when it is bent around a cylindrical R
drum of radius R  24 in. (see figure).

Solution 5.4-1 Steel wire


1 Substitute numerical values:
R  24 in. d in.
16 1/16 in.
From Eq. (5-4): âmax   1300 * 106 ;
2(24 in.) + 1/16 in.
y
âmax 
r
d/2 d
 
R + d/2 2R + d

Problem 5.4-2 A copper wire having diameter d  3 mm is bent into d = diameter


a circle and held with the ends just touching (see figure). If the maximum
permissible strain in the copper is âmax  0.0024, what is the shortest L = length
length L of wire that can be used?

Solution 5.4-2 Copper wire


d  3 mm âmax  0.0024 From Eq. (5-4):
L y d/2 pd
L  2pr r  âmax   
2p r L/2p L
pd p(3 mm)
Lmin    3.93 m ;
âmax 0.0024

389
05Ch05.qxd 9/24/08 4:59 AM Page 390

390 CHAPTER 5 Stresses in Beams (Basic Topics)

Problem 5.4-3 A 4.5 in. outside diameter polyethylene pipe


designed to carry chemical wastes is placed in a trench and bent
around a quarter-circular 90° bend (see figure). The bent section of
the pipe is 46 ft long.
Determine the maximum compressive strain âmax in the pipe.
90°

Solution 5.4-3 Polyethylene pipe


Angle equals 90° or p/2 radians,
L  length of 90° bend r  r  radius of curvature
L  46 ft  552 in. L 2L y d/2
r  âmax  
d  4.5 in. p/2 p r 2L/p

 a b  6400 * 106
pd p 4.5 in.
2pr pr âmax  ;
L  4L 4 552 in.
4 2

Problem 5.4-4 A cantilever beam AB is loaded by a couple M0 at


its free end (see figure.) The length of the beam is L  1.5 m and the A d
longitudinal normal strain at the top surface is 0.001. The distance
from the top surface of the beam to the neutral surface is 75 mm. B
Calculate the radius of curvature r, the curvature k, and the vertical L M0
deflection d at the end of the beam.

Solution 5.4-4
NUMERICAL DATA Deflection: constant curvature for pure bending so
L  2.0 m âmax  0.0012 gives a circular arc; assume flat deflection curve
(small defl.) so BC  L
c  82.5 mm
u  asina b
L L
RADIUS OF CURVATURE sin(u) 
c r r
r r  68.8 m ;
âmax L
u  0.029 radians  0.029
CURVATURE r
1 1  cos(u)  4.232 * 104 r  6.875 * 104 mm
k k  1.455 * 105 m1 ;
r d  r (1  cos(u)) d  29.1 mm ;
05Ch05.qxd 9/24/08 4:59 AM Page 391

SECTION 5.4 Longitudinal Strains in Beams 391

Problem 5.4-5 A thin strip of steel of length L  20 in. and thickness M0 M0


t  0.2 in. is bent by couples M0 (see figure). The deflection at the
midpoint of the strip (measured from a line joining its end points) is found
to be 0.20 in. t d
Determine the longitudinal normal strain â at the top surface of the strip.
L L
— —
2 2

Solution 5.4-5
NUMERICAL DATA d
solving for r: r
b
L  28 inches t  0.25 inches L
1  cosa
d  0.20 inches 2r
0.20
insert numerical data: r
LONGITUDINAL NORMAL STRAIN AT TOP SURFACE
1 cos a b
14
t r
2 t
â â numerical solution for radius of curvature r gives
r 2r r  489.719 inches
L
strain at top (compressive):
2 L t
d  r (1  cos(u)) sin(u)  sin(u)  â â  2.552 * 104
r 2r 2r

assume angle is small so that â  255 11062 ;

d  r a1  cosa bb
L L
u
2r 2r

h d
Problem 5.4-6 A bar of rectangular cross section is loaded and P P
supported as shown in the figure. The distance between supports is
L  1.5 m and the height of the bar is h  120 mm. The deflection
at the midpoint is measured as 3.0 mm.
What is the maximum normal strain â at the top and bottom of
the bar? L L
a — — a
2 2
05Ch05.qxd 9/24/08 4:59 AM Page 392

392 CHAPTER 5 Stresses in Beams (Basic Topics)

Solution 5.4-6
NUMERICAL DATA
d  r a 1  cosa bb
L
L  1.5 m h  120 mm 2r
d  3.0 mm
‹ r a1  cosa b b d  0
L
2r
NORMAL STRAIN AT TOP OF BAR:
numerical solution for radius of curvature r gives
h r  93.749 m
2 h
â â tensile strain, r  radius of strain at top (compressive):
r 2r curvature h
â â  640 * 106 ;
2r
SMALL DEFLECTION SO SMALL ANGLE u
L
2 L
sin(u)  u
r 2r

Normal Stresses in Beams 3


t = — in.
32
Problem 5.5-1 A thin strip of hard copper (E  16,000 ksi) having length
L  90 in. and thickness t  3/32 in. is bent into a circle and held with the
ends just touching (see figure).
(a) Calculate the maximum bending stress smax in the strip.
(b) By what percent does the stress increase or decrease if the thickness of
the strip is increased by 1/32 in.?

Solution 5.5-1
(a) MAXIMUM BENDING STREES smaxnew  69.813 ksi
3 smaxnew  smax
E  16000 ksi L  90 inches t inches (100)  33.3 ;
32 smax
t 33% increase (linear) in max.stress due to increase
sEP Q
2 L in t; same as % increase in thickness t
r
r 2p 4 3

Et 32 32
r  14.324 inches smax  (100)  33.3
2r 3
32
smax  52.4 ksi ;

(b) % CHANGE IN STRESS


4 Etnew
tnew  smaxnew 
32 2r
05Ch05.qxd 9/24/08 4:59 AM Page 393

SECTION 5.5 Normal Stresses in Beams 393

Problem 5.5-2 A steel wire (E  200 GPa) of diameter d  1.25 mm


is bent around a pulley of radius R0  500 mm (see figure).
(a) What is the maximum stress smax in the wire?
(b) By what percent does the stress increase or decrease if the radius of the pulley is
increased by 25%?

R0
d

Solution 5.5-2
(a) MAX. NORMAL STRESS IN WIRE (b) % CHANGE IN MAX. STRESS DUE TO INCREASE
IN PULLEY RADIUS BY 25%
E  200 GPa d  1.25 mm R0  500 mm
d d d
E E E
2 2 2
s smax  snew  snew  199.8 MPa
r d d
R0 + 1.25 R0 +
2 2
snew  smax
smax  250 MPa ; (100)  20% ;
smax

L = length
Problem 5.5-3 A thin, high-strength steel rule (E  30  106 psi)
having thickness t  0.175 in. and length L  48 in. is bent by couples
M0 into a circular are subtending a central angle a  40° (see figure). t
M0 M0
(a) What is the maximum bending stress smax in the rule?
(b) By what percent does the stress increase or decrease if the central angle is
increased by 10%? a

Solution 5.5-3
(a) MAX. BENDING STRESS (b) % CHANGE IN STRESS DUE TO 10% INCREASE
IN ANGLE a
a  40 a b
p
a  0.698 radians E t (1.1a)
180 snew  snew  41997 psi
2L
L  48 inches t  0.175 in. E  30 (106) psi
snew  smax
L (100)  10% ;
r r  68.755 inches smax
a
t linear increase (%)
E
2 Et Eta
smax  smax  smax 
r 2r 2L
smax  38.2 ksi ;
05Ch05.qxd 9/24/08 4:59 AM Page 394

394 CHAPTER 5 Stresses in Beams (Basic Topics)

Problem 5.5-4 A simply supported wood beam AB with span length q


L  4 m carries a uniform load of intensity q  5.8 kN/m (see figure).
(a) Calculate the maximum bending stress smax due to the load q if the A B h
beam has a rectangular cross section with width b  140 mm and
height h  240 mm.
(b) Repeat (a) but use the trapezoidal distuibuted load shown in the b
L
figure part (b).
(a)

q q

2
A B

(b)

Solution 5.5-4
(a) MAX. BENDING STRESS DUE TO UNIFORM LOAD q (b) MAX. BENDING STRESS DUE TO TRAPEZOIDAL LOAD q
2
RA  c a bL + a b Ld
qL I 1 q 1 q1
Mmax  S
8 h 2 2 3 22
2 uniform load (q/2) & triang. load (q/2)
3
bh 1
12 1 RA  qL
S S  bh2 3
h 6
find x  location of zero shear
2
x a bx  0
qL2 q 1 x q
RA 
Mmax 8 2 2 L2
smax  smax 
3x2 + 6Lx  4L2  0
a bh2 b
S 1
6 L  1184 L22
6
3 L2 x
smax  q 2 2(3)
4 bh
 a 1 + 184b
x 1
kN
q  5.8 L4m b  140 mm L 6
m
xmax  0.52753 L
h  240 mm
qL2
Mmax 
8
Mmax  11.6 kN # m
smax  8.63 MPa ;
05Ch05.qxd 9/24/08 4:59 AM Page 395

SECTION 5.5 Normal Stresses in Beams 395

q xmax2 1 xmax q xmax2 Mmax


Mmax  RAxmax   a b smax 
2 2 2 L 2 3 S
Mmax  9.40376 * 102 qL2 N
smax  6.493 * 103 smax  6.49 MPa ;
Mmax  8.727 kN # m m2

Problem 5.5-5 Each girder of the lift bridge (see figure) is


180 ft long and simply supported at the ends. The design load
for each girder is a uniform load of intensity 1.6 k/ft. The girders
are fabricated by welding tree steel plates so as to form an
I-shaped cross section (see figure) having section modulus
S  3600 in.3.
What is the maximum bending stress smax in a girder due
to the uniform load?

Solution 5.5-5 Bridge girder


L  180 ft q  1.6 k/ft
S  3600 in. 3

qL2
Mmax 
8
Mmax qL2
smax  
S 8S
(1.6 k/ft)(180 ft)2(12 in./ft)
smax   21.6 ksi ;
8(3600 in.3)
05Ch05.qxd 9/24/08 4:59 AM Page 396

396 CHAPTER 5 Stresses in Beams (Basic Topics)

Problem 5.5-6 A freight-car axle AB is loaded approximately as shown in P P


the figure, with the forces P representing the car loads (transmitted to the axle
A B
through the axle boxes) and the forces R representing the rail loads (transmitted d
to the axle through the wheels). The diameter of the axle is d  80 mm, the d
distance between centers of the rails is L, and the distance between the forces R R
P and is R is b  200 mm. b b
L
Calculate the maximum bending stress smax in the axle if P  47 kN.

Solution 5.5-6
NUMERICAL DATA MAX. BENDING STRESS
d  82 mm b  220 mm Md
smax 
P  50 kN 2I
pd 4 smax  203 MPa ;
I I  2.219 * 106 m4
64
Mmax  Pb Mmax  11 kN # m

Problem 5.5-7 A seesaw weighing 3 lb/ft of length is occupied by two


children, each weighing 90 lb (see figure). The center of gravity of each
child is 8 ft from the fulcrum. The board is 19 ft long, 8 in. wide, and
1.5 in. thick.
What is the maximum bending stress in the board?

Solution 5.5-7 Seesaw


b  8 in. h  1.5 in.
q  3 lb/ft P  90 lb d  8.0 ft L  9.5 ft
2
qL
Mmax  Pd +  720 lb-ft + 135.4 lb-ft
2
 855.4 lb-ft  10,264 lb-in.
2
bh
S  3.0 in.3.
6
M 10,264 lb-in.
smax    3420 psi ;
S 3.0 in.3
05Ch05.qxd 9/24/08 4:59 AM Page 397

SECTION 5.5 Normal Stresses in Beams 397

Problem 5.5-8 During construction of a highway bridge, 52 mm


the main girders are cantilevered outward from one pier toward
the next (see figure). Each girder has a cantilever length of
48 m and an I-shaped cross section with dimensions shown
in the figure. The load on each girder (during construction)
is assumed to be 9.5 kN/m, which includes the weight of
the girder. 2600 mm
Determine the maximum bending stress in a girder due to 28 mm
this load.

620 mm

Solution 5.5-8
NUMERICAL DATA
Mmax  qL a b
L
Mmax  1.094 * 104 kN m
tf  52 mm tw  28 mm 2
Mmax h
h  2600 mm bf  620 mm smax 
2I
kN
L  48 m q  9.5 smax  101 MPa ;
m
1 1
I (b ) h3  (b  tw) [ h  2 (tf)]3
12 f 12 f
I  1.41 * 1011 mm4

Problem 5.5-9 The horizontal beam ABC of an oil-well pump


has the cross section shown in the figure. If the vertical pumping force acting at
end C is 9 k and if the distance from the line of action of that force to point B is
16 ft, what is the maximum bending stress in the beam due to the pumping
force?

Horizontal beam transfers loads as part of oil well pump

C B A 0.875 in.

22 in.
0.625
in.

8.0 in.
05Ch05.qxd 9/24/08 4:59 AM Page 398

398 CHAPTER 5 Stresses in Beams (Basic Topics)

Solution 5.5-9
NUMERICAL DATA MAX. BENDING STRESS AT B
FC  9 k BC  16 ft
Mmax (12) a b
22
Mmax  F C (BC) Mmax  144 k-ft 2
smax 
I
1 1
I (8) (22)3  (8  0.625) smax  9.53 ksi ;
12 12
* [22  2 (0.875)] 3
I  1.995 * 10 in.
3 4

Problem 5.5-10 A railroad tie (or sleeper) is subjected to two rail P P


loads, each of magnitude P  175 kN, acting as shown in the figure. a L a b
The reaction q of the ballast is assumed to be uniformly distributed
over the length of the tie, which has cross-sectional dimensions h
b  300 mm and h  250 mm.
Calculate the maximum bending stress smax in the tie due to q
the loads P, assuming the distance L  1500 mm and the overhang
length a  500 mm.

Solution 5.5-10 Railroad tie (or sleeper)


DATA P  175 kN b  300 mm h  250 mm Substitute numerical values:
L  1500 mm a  500 mm M1  17,500 N # m M2  21,875 N # m
2P bh2 Mmax  21,875 N # m
q S  3.125 * 103 m3
L + 2a 6
MAXIMUM BENDING STRESS
BENDING-MOMENT DIAGRAM Mmax 21,875 N # m
smax    7.0 MPa ;
5 3.125 * 103 m3
(Tension on top; compression on bottom)

qa2 Pa2
M1  
2 L + 2a
2
a + ab 
q L PL
M2 
2 2 2
2
a + ab 
P L PL

L + 2a 2 2
P
 (2a  L)
4
05Ch05.qxd 9/24/08 4:59 AM Page 399

SECTION 5.5 Normal Stresses in Beams 399

Problem 5.5-11 A fiberglass pipe is lifted by a sling, as shown in the


figure. The outer diameter of the pipe is 6.0 in., its thickness is 0.25 in.,
and its weight density is 0.053 lb/in.3 The length of the pipe is L  36 ft
and the distance between lifting points is s  11 ft.
Determine the maximum bending stress in the pipe due to its own
weight.

s
L

Solution 5.5-11 Pipe lifted by a sling

L  36 ft  432 in. d2  6.0 in. t  0.25 in. p 4


I (d  d14)  18.699 in.4
64 2
s  11 ft  132 in. d1  d2  2t  5.5 in.
p q  gA  (0.053 lb/in.3)(4.5160 in.2)  0.23935 lb/in.
g  0.053 lb/in.3 A  (d22  d21)  4.5160 in.2
4
MAXIMUM BENDING STRESS
a  (L  s)/2  150 in.
Mmax c d2
smax  c  3.0 in.
BENDING-MOMENT DIAGRAM I 2
(2,692.7 lb-in.)(3.0 in.)
smax   432 psi ;
18.699 in.4
(Tension on top)
qa2
M1    2,692.7 lb-in.
2

a  sb  2,171.4 lb-in.
qL L
M2  
4 2
Mmax  2,692.7 lb-in.
05Ch05.qxd 9/24/08 4:59 AM Page 400

400 CHAPTER 5 Stresses in Beams (Basic Topics)

Problem 5.5-12 A small dam of height h  2.0 m is constructed of A


vertical wood beams AB of thickness t  120 mm, as shown in the figure.
Consider the beams to be simply supported at the top and bottom.
Determine the maximum bending stress smax in the beams, assuming
that the weight density of water is g  9.81 kN.m3
h
t

Solution 5.5-12 Vertical wood beam


MAXIMUM BENDING MOMENT

q0 L
RA 
6
q0 x 3
M  RAx 
6L
q0 Lx q0 x 3
h  2.0 m  
6 6L
t  120 mm dM q0L q0x 2 L
  0 x
g  9.81kN/ m3(water) dx 6 2L 13
Let b = width of beam perpendicular to the plane Substitute x  L/13 into the equation for M:
of the figure
L3 q0 L2
a b  a b
q0 L L q0
Let q0 = maximum intensity of distributed load Mmax 
6 13 6L 313 9 13
bt2
q0  gbh S
6 q0 h 2
For the vertical wood beam: L  h; Mmax 
913
Maximum bending stress
Mmax 2q0 h2 2gh3
smax   
S 3 13 bt2 313 t2
SUBSTITUTE NUMERICAL VALUES:
smax  2.10 MPa ;
NOTE: For b  1.0 m, we obtain q0  19,620 N/m,
S  0.0024 m3,
Mmax  5,034.5 N # m, and smax  Mmax/S  2.10 MPa
05Ch05.qxd 9/24/08 4:59 AM Page 401

SECTION 5.5 Normal Stresses in Beams 401

Problem 5.5-13 Determine the maximum tensile stress st y


(due to pure bending about a horizontal axis through C x x
by positive bending moments M) for beams having cross b1 xc
sections as follows (see figure).
xc C xc
C C a a
(a) A semicircle of diameter d h y
(b) An isosceles trapezoid with bases b1  b and b2  4b/3, r
d b2 x
and altitude h O
(c) A circular sector with   p/3 and r  d/2 (b) (c)
(a)

Solution 5.5-13
MAX. TENSILE STRESS DUE TO POSITIVE BENDING MOMENT r4
IS ON BOTTOM OF BEAM CROSS-SECTION Ix  (a + sin (a) cos(a))
4
(a) SEMICIRCLE
a b
2r sin (a)
From Appendix D, Case 10: ybar  c  ybar
3 a
(9p2  64)r4 (9p2  64)d4 d1
Ic  
72p 1152p d 2 p
For a  p/3, r  d/2: A a b a b
4r 2d 2 3
c 
3p 3p
A  d2 a b
p
A  0.2618 d2
Mc 768M M 12
st    30.93 3 ;
(9p  64)d
2 3
2a b sin a b
Ic d d p

± ≤
2 3
(b) ISOSCELES TRAPEZOID c c  0.276 d
3 p
From Appendix D, Case 8: 3
h3(b21 + 4b1b2 + b22) d 4
IC  a b
36(b1 + b2)
a + sina b cosa b b
2 p p p
Ix 
73bh3 4 3 3 3

756 Ix  0.02313 d 4
h(2b1 + b2) 10h IC  Ix  A y2bar
c 
3(b1 + b2) 21 (4p 313) d 13 2
IC  cd 4 d 2 a 12 b c a bd d
p
Mc 360M 768 2 p
st   ;
Ic 73bh2 IC  3.234 * 103 d 4

(c) CIRCULAR SECTOR WITH a  p/3, r  d/2 Mc M


max. tensile stress st  st  85.24 ;
IC d3
From Appendix D, Case 13:
A  r 2 (a)

Problem 5.5-14 Determine the maximum bending stress smax


(due to pure bending by a moment M) for a beam having a cross C b
section in the form of a circular core (see figure). The circle has b
diameter d and the angle b  60°. (Hint: Use the formulas given
in Appendix D, Cases 9 and 15.)
d
05Ch05.qxd 9/24/08 4:59 AM Page 402

402 CHAPTER 5 Stresses in Beams (Basic Topics)

Solution 5.5-14 Circular core


From Appendix D, Cases 9 and 15: pd 4 d4 p
a  b + sin 4b b
1
4 4 3  
 aa  2 + 4 b
pr r ab 2ab 64 32 2 4
Iy 
4 2 r r d4
 (4b  sin4b)
d p 128
r a b
2 2
MAXIMUM BENDING STRESS
b  radians a  radians a  r sin b b  r cos b Mc d
smax  c  r sin b  sin b
4 4 Iy 2
a  b  sin b cos b + 2 sin b cos3 b b
pd d p
Iy  
64 32 2 64M sin b
smax  ;
4 4 d (4b  sin 4b)
3
a  b  (sin b cos b)(1  2 cos2 b)b
pd d p
 
64 32 2 For b  60°  p/3 rad:
pd 4 d4 p
a  b  a sin 2b b(cos 2b)b
1 576M M
  smax   10.96 3 ;
64 32 2 2 (8p 13 + 9)d 3
d

P P
Problem 5.5-15 A simple beam AB of span length L  24 ft d
is subjected to two wheel loads acting at distance d = 5 ft apart
(see figure). Each wheel transmits a load P = 3.0 k, and the A B C
carriage may occupy any position on the beam.
Determine the maximum bending stress smax due to the wheel
loads if the beam is an I-beam having section modulus S  16.2 in.3 L

Solution 5.5-15 Wheel loads on a beam


Substitute x into the equation for M:
d 2
aL  b
P
Mmax 
2L 2
L  24 ft  288 in.
d  5 ft  60 in. MAXIMUM BENDING STRESS
P3k d 2
aL  b
Mmax P
smax   ;
S  16.2 in.3
S 2LS 2
MAXIMUM BENDING MOMENT Substitute numerical values:
P P P 3k
RA  L  x + (L  x  d)  (2L  d  2x) smax  (288 in.  30 in.)2
L L L 2(288 in.)(16.2 in.3)
P  21.4 ksi ;
M  RA x  (2L x  dx  2x )
2
L
dM P L d
 (2L  d  4x)  0 x  
dx L 2 4
05Ch05.qxd 9/24/08 4:59 AM Page 403

SECTION 5.5 Normal Stresses in Beams 403

Problem 5.5-16 Determine the maximum tensile stress st and t


maximum compressive stress sc due to the load P acting on the
simple beam AB (see figure). P d
Data are as follows: P  6.2 kN, L  3.2 m, d  1.25 m, h1
b  80 mm, t  25 mm, h  120 mm, and h1  90 mm. A B h

L b

Solution 5.5-16
NUMERICAL DATA MAX. MOMENT & NORMAL STRESSES
P  6.2 kN L  3.2 m Pd (L  d)
Mmax  Mmax  4.7 kN # m
d  1.25 m b  80 mm L
t  25 mm h  120 mm
MAX. COMPRESSIVE STRESS AT TOP (c  c1)
h1  90 mm Mmax c1
Beam cross section properties: centroid and moment sc  sc  61.0 MPa ;
I
of inertia
Af  b (h  h1) Aw  th1 MAX. TENSILE STRESS AT BOTTOM (c  c2)
(h  h1) Mmax c2
+ Af c h  d
h1
Aw st  st  35.4 MPa ;
2 2 I
c1  c1  76 mm
Af + Aw
c2  h  c1 c2  44 mm dist. to C from bottom
1 1
I t h31 + b (h  h1)3
12 12
(h  h1) 2 h1 2
+ Af cc2  d + Aw ac1  b
2 2
I  5879395.2 mm4

250 lb
Problem 5.5-17 A cantilever beam AB, loaded by a uniform load and a
concentrated load (see figure), is constructed of a channel section. 22.5 lb/ft
Find the maximum tensile stress st and maximum compressive stress sc
if the cross section has the dimensions indicated and the moment of inertia
about the z axis (the neutal axis) is I  3.36 in.4 (Note: The uniform load A B
represents the weight of the beam.) 5.0 ft 3.0 ft

y
0.617 in.
z
C 2.269 in.
05Ch05.qxd 9/24/08 4:59 AM Page 404

404 CHAPTER 5 Stresses in Beams (Basic Topics)

Solution 5.5-17
NUMERICAL DATA MAXIMUM STRESSES
I  3.36 in. 4
c1  0.617 in. MAmax c1
st  st  4341 psi ;
c2  2.269 in. I

22.5 (8)2 MAmax c2


MAmax  + 250 (5) ft-lb sc  sc  15964 psi ;
2 I

MAmax  1970 ft-lb


MAmax (12)  23640 in.-lb

q
Problem 5.5-18 A cantilever beam AB of isosceles trapezoidal b1
cross section has length L  0.8 m, dimensions b1  80 mm,
b2  90 mm, and height h  110 mm (see figure). The beam is C
h
made of brass weighing 85 kN/m3. L
(a) Determine the maximum tensile stress st and maximum b2
compressive stress sc due to the beam’s own weight.
(b) If the width b1 is doubled, what happens to the stresses?
(c) If the height h is doubled, what happens to the stresses?

Solution 5.5-18
NUMERICAL DATA MAX. TENSILE STRESS AT SUPPORT (TOP)
kN Mmax (h  ybar)
L  0.8 m g  85 3
st  st  1.514 MPa ;
m I
b1  80 mm b 2  90 mm
MAX. COMPRESSIVE STRESS AT SUPPORT (BOTTOM)
h  110 mm
Mmax ybar
sc  sc  1.456 MPa ;
(a) MAX. STRESSES DUE TO BEAM’S OWN WEIGHT I
q L2 1
Mmax  q  gA A (b + b 2) h (b) DOUBLE b1& RECOMPUTE STRESSES
2 2 1
b1  160 mm
A  9.35 * 103 mm2
1
N A (b + b2) h A  1.375 * 104 mm2
q  7.9475 * 102 2 1
m
N
Mmax  254.32 N # m q  gA q  1.169 * 103
m
h (2b1 b2)
ybar  ybar  53.922 mm qL2
3 (b1  b2) Mmax 
2
1 b21 4 b1 b2  b222 Mmax  374 N # m
I  h3
36 (b1 b2)
h (2 b1 + b2)
ybar  ybar  60.133 mm
I  9.417 * 10 mm 6 4 3 (b1 + b2)
05Ch05.qxd 9/24/08 4:59 AM Page 405

SECTION 5.5 Normal Stresses in Beams 405

1b21 + 4 b1 b2 + b222 qL2


I  h3 Mmax  Mmax  508.64 N # m
36 (b1 + b2) 2
I  1.35 * 107 mm4 h (2b1 + b2)
ybar  ybar  107.843 mm
3 (b1 + b2)
MAX. TENSILE STRESS AT SUPPORT (TOP)
(b12 + 4b1 b2 + b22)
I  h3
st 
Mmax (h  ybar)
st  1.381 MPa ; 36 1b1 + b22
I
I  7.534 * 107 mm4
MAX. COMPRESSIVE STRESS AT SUPPORT (BOTTOM)
Mmax ybar MAX. TENSILE STRESS AT SUPPORT (TOP)
sc  sc  1.666 MPa ; Mmax (h  ybar)
2 st  st  0.757 MPa ;
I
(c) DOUBLE h & RECOMPUTE STRESSES
b1  80 mm h  220 mm MAX. COMPRESSIVE STRESS AT SUPPORT (BOTTOM)
1 Mmax ybar
A (b + b2) h A  1.87 * 104 mm2 sc  sc  0.728 MPa ;
2 1 I
N
q  gA q  1.589 * 103
m

200 lb/ft
Problem 5.5-19 A beam ABC with an overhang from B to C supports
a uniform load of 200 lb/ft throughout its length (see figure). The beam is
a channel section with dimensions as shown in the figure. The moment of A C
B
inertia about the z axis (the neutral axis) equals 8.13 in.4
Calculate the maximum tensile stress st and maximum compressive
stress sc due to the uniform load. 12 ft 6 ft

y
0.787 in.
z
C 2.613 in.

Solution 5.5-19
NUMERICAL DATA LOCATON OF ZERO SHEAR IN SPAN AB & MAX. (+)
MOMENT IN SPAN AB
lb
q  200 I  8.13 in.4
ft RA
xmax  xmax  4.5 ft
c1  0.787 in. c2  2.613 in. q
xmax2
COMPUTE SUPPORT REACTIONS MmaxAB  RA xmax  q
2
(18)2 MmaxAB  2025 ft-lb
q
2 max. () moment at B
a MA  0 RB 
12
RB  2700 lb
(6)2
MB  q MB  3600 ft-lb
a Fv  0 RA  q (18)  RB R A  900 lb 2
05Ch05.qxd 9/24/08 4:59 AM Page 406

406 CHAPTER 5 Stresses in Beams (Basic Topics)

MAX. STRESSES IN SPAN AB MAX. STRESSES IN SPAN BC


MmaxAB (12) c1 MB (12) c2
sC  sc  2352 psi sc 
I I
MmaxAB (12) c2 sc  13885 psi ; max. compressive stress
st 
I MB (12) c1
st  7810 psi ; max. tensile stress st  st  4182 psi
I

A
Problem 5.5-20 A frame ABC travels horizontally with an acceleration t
a0 (see figure). Obtain a formula for the maximum stress smax in the
vertical arm AB, which had length L, thickness t , and mass density r. a0 = acceleration
L

B C

Solution 5.5-20 Accelerating frame


L  length of vertical arm TYPICAL UNITS FOR USE
t  thickness of vertical arm IN THE PRECEDING EQUATION
r  mass density
SI units: r  kg/m3  N # s2/m4
a0  acceleration
Let b  width of arm perpendicular to the plane of the figure L  meters (m)
Let q  inertia force per unit distance along vertical arm a0  m/s2
VERTICAL ARM t  meters (m)
smax  N/m2 (pascals)

USCS units: r  slug/ft3  lb-s2/ft4


qL2 rbta0L2
q  rbta0 Mmax   L  ft a0  ft/s2 t  ft
2 2
bt2 Mmax 3rL2a0 smax  lb/ft (Divide by 144 to obtain psi)
2

S smax   ;
6 S t
05Ch05.qxd 9/24/08 4:59 AM Page 407

SECTION 5.5 Normal Stresses in Beams 407

Problem 5.5-21 A beam of T-section is supported and 3


t=—
8 in.
loaded as shown in the figure. The cross section has width P = 700 lb
b  2 1/2 in., height h  3 in., and thickness t  3/8 in. L1 = 3 ft q = 100 lb/ft
Determine the maximum tensile and compressive stresses 3
t=—
8 in.
in the beam. h=
3 in.

1
L2 = 8 ft L3 = 5 ft b = 2—
2 in.

Solution 5.5-21
NUMERICAL DATA
a Fv  0 Rlf  P + qL3  Rrt R lf  281 lb
L1  3 ft L2  8 ft L3  5 ft
Moment diagram (843.75 ft-lb at load P, 1250 ft-lb
lb
P  700 lb q  100 at right support)
ft
3 8.438E+02
t in. h  3 in. b  2.5 in.
8
Find centroid of cross section (c2 from bottom,
c1 from top) Aw  t (h  t) Af  t b
ht
+ Aw at + b
t –1.250E+03
Af
2 2
c2  c2  1 in.
Af + Aw
MP  843.75 ft-lb
c1  h  c2 c1  2 in.
Mrt  1250 ft-lb
ht
Aw a b + Af ah  b
t
2 2 MAX. STRESSES IN BEAM
check c1 
Af + Aw at load P
c1  2 c1 + c2  3 equals h MP (12) c1
sc  sc  12494 psi ;
MOMENT OF INERTIA I
(max. compressive stress)
t 2
b t 3 + Af ac2  b
1 1 MP (12) c2
I t (h  t)3 + st  st  5842 psi
12 12 2 I
(h  t) 2
+ Aw cc1  d I  2 in.4 at right support
2 Mrt (12) c2
sc  sc  8654 psi
FIND SUPPORT REACTIONS-SUM MOMENTS ABOUT LEFT I
SUPPORT Mrt (12) c1
st  st  18509 psi ;
PL1 + qL3 aL2 + b
L3 I
2 (max. tensile stress)
a Mlf  0 Rrt 
L2
Rrt  919 lb
05Ch05.qxd 9/24/08 4:59 AM Page 408

408 CHAPTER 5 Stresses in Beams (Basic Topics)

Problem 5.5-22 A cantilever beam AB with a rectangular 10 mm


cross section has a longitudinal hole drilled throughout its length 50 mm
(see figure). The beam supports a load P  600 N. The cross A B 12.5 mm
section is 25 mm wide and 50 mm high, and the hole has a
diameter of 10 mm.
Find the bending stresses at the top of the beam, at the top P = 600 N 37.5 mm
of the hole, and at the bottom of the beam.
L = 0.4 m

25 mm

Solution 5.5-22 Rectangular beam with a hole


MOMENT OF INERTIA ABOUT THE NEUTRAL AXIS
(THE z AXIS)
All dimensions in millimeters.
Rectangle:
Iz  Ic + Ad2
1
MAXIMUM BENDING MOMENT  (25)(50)3 + (25)(50)(25  24.162)2
12
M  PL  (600 N)(0.4 m)  240 N # m  260,420 + 878  261,300 mm4

PROPERTIES OF THE CROSS SECTION Hole:


p
A1  area of rectangle Iz  Ic + Ad2  (10)4 + (78.54)(37.5  24.162)2
64
 (25 mm)(50 mm)  1250 mm2
 490.87 + 13,972  14,460 mm4
A2  area of hole
Cross-section:
p I  261,300  14,460  246,800 mm4
 (10 mm)2  78.54 mm2
4
STRESS AT THE TOP OF THE BEAM
A  area of cross section
Mc1 (240 N # m)(25.838 mm)
 A1  A2  1171.5 mm s1  
I 246,800 mm4
Using line B  B as reference axis:  25.1 MPa ;
©Aiyi  A1(25 mm)  A2(37.5 mm)  28,305 mm3 (tension)
STRESS AT THE TOP OF THE HOLE
y a
Aiyi 28,305 mm3
  24.162 mm My
A 1171.5 mm2 s2  y  c1  7.5 mm  18.338 mm
I
Distances to the centroid C: (240 N # m)(18.338 mm)
c2  y  24.162 mm s2   17.8 MPa ;
246,800 mm4
c1  50 mm  c2  25.838 mm (tension)
STRESS AT THE BOTTOM OF THE BEAM
Mc2 (240 N # m)(24.162 mm)
s3   
I 246,800 mm4
 23.5 MPa ;
(compression)
05Ch05.qxd 9/24/08 4:59 AM Page 409

SECTION 5.5 Normal Stresses in Beams 409

Problem 5.5-23 A small dam of height h  6 ft is constructed of Steel beam


vertical wood beams AB, as shown in the figure. The wood beams, A Wood beam
t
which have thickness t  2.5 in., are simply supported by horizontal
steel beams at A and B. t
Construct a graph showing the maximum bending stress smax in Steel beam Wood beam
the wood beams versus the depth d of the water above the lower
support at B. Plot the stress smax (psi) as the ordinate and the h
depth d (ft) as the abscissa. (Note: The weight density g of water d
equals 62.4 lb/ft3.)
B

Side view Top view

Solution 5.5-23 Vertical wood beam in a dam


h  6 ft MAXIMUM BENDING STRESS
1
t  2.5 in. Section modulus: S  bt2
6
g  62.4 lb/ft3
6 q0d2
 2c a1  + bd
Mmax d 2d d
Let b  width of beam smax 
S bt 6 L 3L A 3L
(perpendicular to the
figure) q0  g bd
Let q0  intensity of gd3
a1  b
d 2d d
load at depth d smax  + ;
t2 L 3L A 3L
q0  gbd
SUBSTITUTE NUMERICAL VALUES:
ANALYSIS OF BEAM
d  depth of water (ft) (Max. d  h  6 ft)
L  h  6 ft
q0d2 L  h  6 ft g  62.4 lb/ ft3 t  2.5 in.
RA  smax  psi
6L
(62.4)d3
a3  b a1  b
q0d d d d d
RB  smax  +
6 L (2.5)2 6 9 A 18
d  0.1849d3(54  9d + d12d ) ;
x0  d
A 3L d(ft) smax(psi)
q0d2
a1  b
d 0 0
Mc  RA(L  d)  1 9
6 L
2 59
3 171
4 347
5 573
6 830

q 0 d2
a1  + b
d 2d d
Mmax 
6 L 3L A 3L
05Ch05.qxd 9/24/08 4:59 AM Page 410

410 CHAPTER 5 Stresses in Beams (Basic Topics)

Problem 5.5-24 Consider the nonprismatic cantilever beam q0


of circular cross section shown. The beam has an internal cylindrical y
Linea P = q0L/2
hole in segment 1; the bar is solid (radius r) in segment 2. The beam r q(x
)
is loaded by a downward triangular load with maximum intensity q0
as shown. M1 x
Find expressions for maximum tensile and compressive flexural
stresses at joint 1. 1 2 3
2L L
R1 — —
3 3
Segment 1 Segment 2
0.5 EI EI

Solution 5.5-24
STATICS MAX. STRESSES AT JOINT 1

q0 a b 
1 2L q0 L MAX. COMPRESSION AT TOP (RADIUS r)
a Fv  0 R1 
2 3 2 23
q L2 (r)
1 M1 r 54 0
R1  qL sc  sc 
6 0 0.5 EI EI
2
g M1  0
23 q0 L2 r 23
M1  c q0 a b a b  Ld
1 2L 1 2L q0 L sc  ;  0.852
27 EI 27
2 3 3 3 2
Max. tensile stress at bottom  same magnitude as
23 23
M1  q0 L2  0.426 compressive stress at top
54 54

Problem 5.5-25 A steel post (E  30  106 psi) having thickness t  1/8 in.
and height L  72 in. supports a stop sign (see figure: s  12.5 in.). The height
of the post L is measured from the base to the centroid of the sign. The stop sign
is subjected to wind pressure p  20 lb/ft2 normal to its surface. Assume that
the post is fixed at its base.
(a) What is the resultant load on the sign? [See Appendix D, Case 25, for
properties of an octagon, n  8].
(b) What is the maximum bending stress smax in the post?
05Ch05.qxd 9/24/08 4:59 AM Page 411

SECTION 5.5 Normal Stresses in Beams 411

y Circular cutout, d = 0.375 in.


Section A–A 5/8 in.
Post, t = 0.125 in.

c1
z
1.5 in.
C
c2

Stop sign
0.5 in. 1.0 in. 1.0 in. 0.5 in.

Wind load

Numerical properties of post


A = 0.578 in.2, c1 = 0.769 in., c2 = 0.731 in.,
Iy = 0.44867 in.4, Iz = 0.16101 in.4

A A

Elevation
view of post
05Ch05.qxd 9/24/08 4:59 AM Page 412

412 CHAPTER 5 Stresses in Beams (Basic Topics)

Solution 5.5-25
(a) RESULTANT LOAD F ON SIGN (b) MAX. BENDING STRESS IN POST
p  20 psf s  12.5 in. n8 L  72 in. I Z  0.16101 in.4

a b
360 p c1  0.769 in. c2  0.731 in.
b b  0.785 rad
n 180 Mmax
Mmax  FL  628.701 ft-lb
ns2
cota b
b 12
A A  754.442 in.2
4 2 Mmax c1
sc  sc  36.0 ksi ;
or A  5.239 ft2 Iz

F  pA F  104.8 lb ; (max. bending stress at base of post)


Mmax c2
st  st  34.2 ksi
Iz

Design of Beams
P s2 P
Problem 5.6-1 The cross section of a narrow-gage railway Steel rail
bridge is shown in part (a) of the figure. The bridge is Wood
constructed with longitudinal steel girders that support the tie d
wood cross ties. The girders are restrained against lateral
buckling by diagonal bracing, as indicated by the dashed lines. b
The spacing of the girders is s1  50 in. and the spacing
of the rails is s2  30 in. The load transmitted by each rail to a Steel
girder
single tie is P  1500 lb. The cross section of a tie, shown in (b)
part (b) of the figure, has width b  5.0 in. and depth d.
Determine the minimum value of d based upon an s1
allowable bending stress of 1125 psi in the wood tie.
(Disregard the weight of the tie itself.) (a)

Solution 5.6-1 Railway cross tie


P(s1  s2)
Mmax   15,000 lb-in.
2
bd 2 1 5d 2
S  (50 in.)(d 2)  d  inches
6 6 6
5d 2
Mmax  s allow S 15,000  (1125)a b
6
s1  50 in. b  5.0 in. s2  30 in. Solving, d 2  16.0 in. dmin  4.0 in. ;

d  depth of tie P  1500 lb sallow  1125 psi 3P(s1  s2)


NOTE: Symbolic solution: d 2 
bsallow
05Ch05.qxd 9/24/08 4:59 AM Page 413

SECTION 5.6 Design of Beams 413

Problem 5.6-2 A fiberglass bracket ABCD of solid circular cross section has 6b
the shape and dimensions shown in the figure. A vertical load p  40 N acts at
the free end D. A B
Determine the minimum permissible diameter dmin of the bracket if the
allowable bending stress in the material is 30 MPa and b  37 mm.
(Note: Disregard the weight of the bracket itself.) 2b
D C

P 2b

Solution 5.6-2
1 1
(3Pb) a b
dmin
a b c d
96Pb 3 96 (40) (37) 3
2 96Pb dmin dmin
sa  dmin3  psa p (30)
pdmin4 psa
a b dmin  11.47 mm ;
64

P  2750 lb
Problem 5.6-3 A cantilever beam of length L  7.5 ft supports a uniform
load of intensity q  225 lb/ft and a concentrated load P  2750 lb (see figure).
Calculate the required section modulus S if sallow  17,000 psi. Then select a q  225 lb/ft
suitable wide-flange beam (W shape) from Table E-1(a), Appendix E, and recalcu-
late S taking into account the weight of beam. Select a new beam size if necessary.

L = 7.5 ft

Solution 5.6-3
sa  17000 psi P  2750 lb Mmax2  2.774 * 104 lb-ft
lb Mmax2 (12)
q  225 L  7.5 ft smax  smax  13699 psi
ft Sact
qL2 below allowable -OK
Mmax1  PL + Mmax1  2.695 * 104 lb-ft
2
Repeat for W14 * 26 which is lighter than W8 * 28
Find Sreqd without beam weight
lb
Mmax1 (12) w  26 Sact  35.3 in.3
Sreqd  Sreqd  19.026 in. 3 ft
sa (q + w) L2
Mmax3  PL +
try W 8 * 28 (S  24.3 in. )
3
2
Mmax3  2.768 * 104 lb-ft
Check - add weight per ft for beam
M max3 (12)
lb smax  smax  9411 psi
W  28 Sact  24.3 in.3 Sact
ft
(q + w) L2 well below allowable - OK
Mmax2  PL +
2 use W 14 * 26 ;
05Ch05.qxd 9/24/08 4:59 AM Page 414

414 CHAPTER 5 Stresses in Beams (Basic Topics)

Problem 5.6-4 A simple beam of length L  5 m carries a uniform load P = 22.5 kN 1.5 m
kN
of intensity q  5.8 and a concentrated load 22.5 kN (see figure). q = 5.8 kN/m
m
Assuming sallow  110 MPa, calculate the required section modulus S.
Then select an 200 mm wide-flange beam (W shape) from Table E-1(b)
Appendix E, and recalculate S taking into account the weight of beam. Select a
new 200 mm beam if necessary.
L=5m

Solution 5.6-4
NUMERICAL DATA RECOMPUTE MAX. MOMENT WITH BEAM MASS INCLUDED &
THEN CHECK ALLOWABLE STRESS
kN
L  5 m q  5.8
w  a41.7 b a9.81 2 b
m kg M
P  22.5 kN b  1.5 m m s
aLb a  3.5 m N
w  409.077 Sact  398 * 103 mm3
sallow  110 MPa m

aq + bL
qL Pb W
statics RA  + RA  21.25 kN 1000
2 L Pd
RA  +
qL Pa 2 L
RB  + RB  30.25 kN
2 L RA
RA  22.273 kN xm 
qL + P  51.5 kN RA + RB  51.5 kN q + W
xm  3.587 m greater than a so max. moment at load pt
LOCATE POINT OF ZERO SHEAR
(q + W ) a2
RA Mmax  RA a 
xm  xm  3.664 m 2
q
Mmax  39.924 kN # m
greater than dist. a to load P so zero shear is at load
Mmax
point smax 
S act
q a2
Mmax  RA a  Mmax  38.85 kN # m smax  100.311 MPa OK, less than 110 MPa
2
FIND REQUIRED SECTION MODULUS
Mmax
Sreqd  Sreqd  353.182 * 103 mm3
sallow
select W 200 * 41.7 ; (Sact  398 * 103 mm3)

q P q
Problem 5.6-5 A simple beam AB is loaded as shown in the figure.
Calculate the required section modulus S if sallow  17,000 psi, L  28 ft,
P  2200 lb, and q  425 lb/ft. Then select a suitable I-beam (S shape) from A B
Table E-2(a), Appendix E, and recalculate S taking into account the weight of
the beam. Select a new beam size if necessary.
L L L L
— — — —
4 4 4 4
05Ch05.qxd 9/24/08 4:59 AM Page 415

SECTION 5.6 Design of Beams 415

Solution 5.6-5
NUMERICAL DATA RECOMPUTE REACTIONS AND MAX. MOMENT THEN CHECK
lb
sa  17000 psi L  28 ft MAX. STRESS w  25.4
ft
lb
P  2200 lb q  425 P L L
ft RA  + q + w RA  4.431 * 103 lb
FIND REACTIONS (EQUAL DUE TO SYMMETRY) THEN MAX. 2 4 2

a + b w a b
MOMENT AT CENTER OF BEAM L qL L 1L L 1L
Mmax  RA 
P L 2 4 4 24 2 22
RA  + q RA  4.075 * 103 lb
2 4 Mmax  2.83 * 104 ft-lb

a + b
L qL L 1L Mmax (12)
Mmax  RA  smax 
2 4 4 24 Sact
Mmax  2.581 * 104 ft-lb smax  13,806 psi less than allowable so OK
Compute Sreqd & then select S shape
Mmax (12)
Sreqd  Sreqd  18.221 in.3
sa
select S 10 * 25.4 ;
(Sact  24.6 in. , w  25.4 lb/ft)
3

Problem 5.6-6 A pontoon bridge (see figure) is constructed of two Chess


longitudinal wood beams, known an balks, that span between adjacent
Pontoon
pontoons and support the transverse floor beams, which are called
chesses.
For purposes of design, assume that a uniform floor load of
8.0 kPa acts over the chesses. (This load includes an allowance for
the weights of the chesses and balks.) Also, assume that the chesses
are 2.0 m long and that the balks are simply supported with a span
of 3.0 m. The allowable bending stress in the wood is 16 MPa. Balk
If the balks have a square cross section, what is their
minimum required width bmin?

Solution 5.6-6 Pontoon bridge

FLOOR LOAD: W  8.0 kPa

ALLOWABLE STRESS: sallow  16 MPa


Lc  length of chesses Lb  length of balks
 2.0 m  3.0 m
05Ch05.qxd 9/24/08 4:59 AM Page 416

416 CHAPTER 5 Stresses in Beams (Basic Topics)

LOADING DIAGRAM FOR ONE BALK b3


Section modulus S 
6
qL2b (8.0 kN/m)(3.0 m)2
Mmax    9,000 N # m
8 8
Mmax 9,000 N # m
S   562.5 * 106 m3
sallow 16 MPa

W  total load b3
‹  562.5 * 106 m3 and b3  3375 * 106 m3
6
 wLbLc
Solving, bmin  0.150 m  150 mm ;
W wLc
q 
2Lb 2
(8.0 kPa)(2.0 m)

2
 8.0 kN/m

Problem 5.6-7 A floor system in a small building consists of


wood planks supported by 2 in. (nominal width) joists spaced at
distance s, measured from center to center (see figure). The span Planks
length L of each joist is 10.5 ft, the spacing s of the joists is 16 in.,
and the allowable bending stress in the wood is 1350 psi. The uni-
form floor load is 120 lb/ft2, which includes an
allowance for the weight of the floor system itself.
Calculate the required section modulus S for the joists, and s
then select a suitable joist size (surfaced lumber) from Appendix F,
s
assuming that each joist may be represented as a simple beam L
carrying a uniform load. s
Joists

Solution 5.6-7 Floor joists


qL2 1
Mmax   (13.333 lb/in.)(126 in.)2  26,460 lb-in.
8 8
Mmax 26,460 lb/in.
Required S    19.6 in.3 ;
sallow 1350 psi
From Appendix F: Select 2 * 10 in. joists ;
sallow  1350 psi
L  10.5 ft  126 in.
w  floor load  120 lb/ft2  0.8333 lb/in.2
s  spacing of joists  16 in.
q  ws  13.333 lb/in.
05Ch05.qxd 9/24/08 4:59 AM Page 417

SECTION 5.6 Design of Beams 417

Problem 5.6-8 The wood joists supporting a plank floor (see figure) are
40 mm * 180 mm in cross section (actual dimensions) and have a span
length L  4.0 m. The floor load is 3.6 kPa, which includes the weight
of the joists and the floor.
Calculate the maximum permissible spacing s of the joists if the
allowable bending stress is 15 MPa. (Assume that each joist may be
represented as a simple beam carrying a uniform load.)

Solution 5.6-8 Spacing of floor joists

L  4.0 m
w  floor load  3.6 kPa sallow  15 MPa
s  spacing of joists

q  ws 4 bh2sallow
2 SPACING OF JOISTS smax  ;
bh 3wL2
S
6 Substitute numerical values:
qL2 wsL2 4(40 mm)(180 mm)2(15 MPa)
Mmax   smax 
8 8
3(3.6 kPa)(4.0 m)2
Mmax 2 2
wsL bh  0.450 m  450 mm ;
S  
sallow 8sallow 6
05Ch05.qxd 9/24/08 4:59 AM Page 418

418 CHAPTER 5 Stresses in Beams (Basic Topics)

Problem 5.6-9 A beam ABC with an overhang from B to C is q0


constructed of a C 10  30 channel section (see figure). The beam
supports its own weight (30 lb/ft) plus a triangular load of maximum C
A B
intensity q0 acting on the overhang. The allowable stresses in tension
and compression are 20 ksi and 11 ksi, respectively.
Determine the allowable triangular load intensity q0,allow if the L L
distance L equals 3.5 ft.

3.033 in. C 2.384 in.


0.649 in.
10.0 in.

Solution 5.6-9
NUMERICAL DATA check tension on top
lb MB c1 I22
w  30 sat  20 ksi sac  11 ksi st  MB  s at
ft I22 c1
L  3.5 ft
csat a b  wL2 d
3 I22 1
q0allow  2
c1  2.384 in. c2  0.649 in. L c1 2
from Table E-3(a) I22  3.93 in. 4 q0allow  628 lb/ft ; governs
MAX. MOMENT IS AT B (TENSION TOP, COMPRESSION BOTTOM) check compression on bottom

csac a b  wL2 d
3 I22 1
+ q0L a Lb
L 1 2 q0allow 
MB  wL 2 c2 2
2 2 3 L
1 1 lb
MB  wL2 + q0L2 q0allow  1314
2 3 ft

Problem 5.6-10 A so-called “trapeze bar” in a hospital


room provides a means for patients to exercise while in bed
(see figure). The bar is 2.1 m long and has a cross section in C
the shape of a regular octagon. The design load is 1.2 kN applied h
at the midpoint of the bar, and the allowable bending stress is
200 MPa.
Determine the minimum height h of the bar. (Assume
that the ends of the bar are simply supported and that the
weight of the bar is negligible.)
05Ch05.qxd 9/24/08 4:59 AM Page 419

SECTION 5.6 Design of Beams 419

Solution 5.6-10 Trapeze bar (regular octagon)

P  1.2 kN L  2.1 m sallow  200 MPa b  0.41421h ‹ Ic  1.85948(0.41421h)4  0.054738h4


Determine minimum height h. SECTION MODULUS
Ic 0.054738h4
MAXIMUM BENDING MOMENT S   0.109476h3
h/2 h/2
PL (1.2 kN)(2.1 m)
Mmax    630 N # m
4 4 MINIMUM HEIGHT h
PROPERTIES OF THE CROSS SECTION M M
s S
Use Appendix D, Case 25, with n  8 S s
630 N # m
b  length of one side 0.109476h3   3.15 * 106 m3
200 MPa
360° 360°
b   45° h3  28.7735 * 106 m3 h  0.030643 m
n 8
‹ hmin  30.6 mm ;
b b
tan  (from triangle)
2 h
ALTERNATIVE SOLUTION (n  8)
b h
cot  PL b b
2 b M b  45° tan  12 1 cot  12 1
4 2 2
b  (12  1)h h  (12 + 1)b
412  5 4
Ic  a bb  a bh
11 + 812 4
b 45° 12 12
For b  45°:  tan  0.41421
h 2
412  5 3
S a bh
3PL
h 45° h3  ;
 cot  2.41421 6 2(4 12  5)sallow
b 2
Substitute numerical values:
MOMENT OF INERTIA h3  28.7735 * 106 m3 hmin  30.643 mm ;
4
acot b a3 cot2 1b
nb b b
Ic 
192 2 2
8b4
Ic  (2.41421)[3(2.41421)2 1]  1.85948b4
192
05Ch05.qxd 9/24/08 4:59 AM Page 420

420 CHAPTER 5 Stresses in Beams (Basic Topics)

Problem 5.6-11 A two-axle carriage that is part of an overhead traveling 3800 lb 2200 lb
crane in a testing laboratory moves slowly across a simple beam AB 5 ft
(see figure). The load transmitted to the beam from the front axle is 2200 lb
and from the rear axle is 3800 lb. The weight of the beam itself may be
disregarded. A B
(a) Determine the minimum required section modulus S for the beam
if the allowable bending stress is 17.0 ksi, the length of the beam is
18 ft
18 ft, and the wheelbase of the carriage is 5 ft.
(b) Select the most economical I-beam (S shape) from Table E-2(a), Appendix E.

Solution 5.6-11
NUMERICAL DATA (P1 + P2) L  P1d
xm  xm  8.083 ft
L  18 ft P1  2200 lb 2 (P1 + P2)
L  xm L  (xm + d)
RA  P2 a b + P1 c d
P2  3800 lb d  5 ft
sa  17 ksi L L
RA  2694 lb
(a) FIND REACTION RA THEN AN EXPRESSION FOR MOMENT
L(xm d)
Mmax  xm cP2 a b P1 c dd
UNDER LARGER LOAD P2; LET X  DIST. FROM A Lxm
TO LOAD P2 L L
Lx L  (x + d ) Mmax  21780 ft-lb
RA  P2 a b + P1 c d
L L Mmax
Sreqd  Sreqd  15.37 in.3 ;
M2  RA x sa

Lx L  (x + d )
M2  x cP2 a b + P1 c dd (b) SELECT MOST ECONOMICAL S SHAPE FROM
L L TABLE E-2(A)
xP2 LP2 x2 xP1LP1x2 xP1d select S8 * 23 ; Sact  16.2 in.3
M2 
L
Take derivative of MA & set to zero to find max.
bending moment at x  x m
d xP2LP2x2 xP1LP1x2 xP1d
a b
dx L
P2L  2P2x + P1L  2P1x  P1d

L
P2L  2P2x + P1L  2P1x  P1d  0

Problem 5.6-12 A cantilever beam AB of circular cross section and length A


L  450 mm supports a load P  400 N acting at the free end (see figure). B
The beam is made of steel with an allowable bending stress of 60 MPa.
Determine the required diameter dmin of the beam, considering the d
effect of the beam’s own weight. P
L
05Ch05.qxd 9/24/08 4:59 AM Page 421

SECTION 5.6 Design of Beams 421

Solution 5.6-12 Cantilever beam


DATA L  450 mm P  400 N MINIMUM DIAMETER
sallow  60 MPa Mmax  sallow S
g  weight density of steel pgd 2L2 pd 3
PL +  sallow a b
 77.0 kN/m 3
8 32
Rearrange the equation:
WEIGHT OF BEAM PER UNIT LENGTH
32 PL
pd 2 sallow d 3  4gL2 d 2  0
q  ga b p
4
(Cubic equation with diameter d as unknown.)
MAXIMUM BENDING MOMENT Substitute numerical values (d  meters):
q L2 pgd3L2 (60 * 106 N/m2)d3  4(77,000 N/m3)(0.45 m)2d2
Mmax  PL +  PL +
2 8
32
 (400 N)(0.45 m)  0
pd 3 p
SECTION MODULUS S
32 60,000d 3  62.37d 2  1.833465  0
Solve the equation numerically:
d  0.031614 m dmin  31.61 mm ;

q
Problem 5.6-13 A compound beam ABCD (see figure) is
supported at points A, B, and D and has a splice at point C.
The distance a  6.25 ft, and the beam is a S 18  70 A
B C D
wide-flange shape with an allowable bending stress of 12,800 psi. Splice (a) (b)
Moment Shear
(a) If the splice is a moment release, find the allowable 4a a 4a release release
uniform load qallow that may be placed on top of the
beam, taking into account the weight of the beam itself.
[See figure part (a).]
(b) Repeat assuming now that the splice is a shear release, as in figure part (b).

Solution 5.6-13
NUMERICAL DATA
MZ 2.000E+00
lb @ 2.000E+00
w  70 S  103 in.3 MZ 9.453E–01
ft
@ 1.375E+00
a  6.25 ft sa  12800 psi
× × ×
(a) MOMENT RELEASE AT C-GIVES MAX. MOMENT AT B
(SEE MOMENT DIAGRAM)  2.5 q a2
Mmax
sa  Mmax  [1qallow + w2 a2 (2.5)]
S MZ –2.500E+00
and Mmax  s a S @ 4.000E+00

lb
w  70 S  103 in.3
ft
a  6.25 ft sa  12800 psi
05Ch05.qxd 9/24/08 4:59 AM Page 422

422 CHAPTER 5 Stresses in Beams (Basic Topics)

sa S MZ 8.00E+00
12 in./ft
qallow  w
2.5 a2
lb
qallow  1055 ; for moment release
ft

(b) SHEAR RELEASE AT C-GIVES MAX. MOMENT AT C


(SEE MOMENT DIAGRAM)  8 q a2
lb
sa S qallow  282 ; for shear release
ft
12 in./ft
qallow  w
8a2

Problem 5.6-14 A small balcony constructed of wood is


supported by three identical cantilever beams (see figure).
Each beam has length L1  2.1 m, width b, and height h  4b/3.
The dimensions of the balcon floor are L1 * L2,
with L2  2.5 m. The design load is 5.5 kPa acting over the
entire floor area. (This load accounts for all loads except
the weights of the cantilever beams, which have a weight 4b
density g  5.5 kN/m3.) The allowable bending stress in the h= —
3
cantilevers is 15 MPa.
Assuming that the middle cantilever supports 50% of the L2 L1 b
load and each outer cantilever supports 25% of the load,
determine the required dimensions b and h.

Solution 5.6-14 Compound beam


MAXIMUM BENDING MOMENT
(qq0)L21 1
Mmax   (6875 N/m7333b2)(2.1 m)2
2 2
 15,159 + 16,170b2 (N # m)
bh2 8b3
L1  2.1 m L2  2.5 m Floor dimensions: L1 * L2 S 
6 27
Design load  w  5.5 kPa Mmax  sallow S
g  5.5 kN/m3 (weight density of wood beam)
8b3
sallow  15 MPa 15,159 + 16,170b2  (15 * 106 N/m2)a b
27
MIDDLE BEAM SUPPORTS 50% OF THE LOAD. Rearrange the equation:

b  (5.5 kPA) a b  6875 N/m


L2 2.5 m (120 * 106)b3  436,590b2  409,300  0
‹ q  wa
2 2
SOLVE NUMERICALLY FOR DIMENSION b
WEIGHT OF BEAM 4b
b  0.1517 m h  0.2023 m
4gb2 4 3
q0  gbh   (5.5 kN/m2) b2
3 3
REQUIRED DIMENSIONS
 7333b2 (N/m) (b  meters)
b  152 mm h  202 mm ;
05Ch05.qxd 9/24/08 4:59 AM Page 423

SECTION 5.6 Design of Beams 423

Problem 5.6-15 A beam having a cross section in the form of an y


unsymmetric wide-flange shape (see figure) is subjected to a negative b
bending moment acting about the z axis.
Determine the width b of the top flange in order that the stresses at 1.5 in.
the top and bottom of the beam will be in the ratio 4:3, respectively. 1.25 in.
z 12 in.
C

1.5 in.
16 in.

Solution 5.6-15 Unsymmetric wide-flange beam


AREAS OF THE CROSS SECTION (in.2)
A1  1.5b A2  (12)(1.25)  15 in.2
A3  (16)(1.5)  24 in.2
A  A1 + A2 + A3  39 + 1.5b (in.2)

FIRST MOMENT OF THE CROSS-SECTIONAL


AREA ABOUT THE LOWER EDGE B-B

QBB  gyi Ai  (14.25)(1.5b) + (7.5)(15) + (0.75)(24)


Stresses at top and bottom are in the ratio 4:3.  130.5 + 21.375b (in.3)
Find b (inches)
h  height of beam  15 in. DISTANCE c2 FROM LINE B-B TO THE CENTROID C
QBB 130.5 + 21.375b 45
LOCATE CENTROID c2    in.
A 39 + 1.5b 7
stop c1 4
 
sbottom c2 3 SOLVE FOR b
4 60
c1  h   8.57143 in. (39 + 1.5b)(45)  (130.5 + 21.375b)(7)
7 7
3 45 82.125b  841.5 b  10.25 in. ;
c2  h   6.42857 in.
7 7

Problem 5.6-16 A beam having a cross section in the form of a channel y


(see figure) is subjected to a bending moment acting about the z axis.
Calculate the thickness t of the channel in order that the bending stresses
at the top and bottom of the beam will be in the ratio 7:3, respectively. t t

t 55 mm
z C

152 mm
05Ch05.qxd 9/24/08 4:59 AM Page 424

424 CHAPTER 5 Stresses in Beams (Basic Topics)

Solution 5.6-16
NUMERICAL DATA ratio of top to bottom stresses  c1/c2  7/3
h  152 mm b  55 mm 1 11385  186 t + t2
take 1st moments to find distances c1 & c2 2  131 + t

(152  2t) (t) + 2.55 t a b


1st moments about base t 55

J K
2 2
(h  2t) (t) + 2bt a b
t b
2 2 2.55 t + t (152  2 t)
c2 
2bt + t (h  2t) 111385  186 t + t22
  7/3
c1  b  c2  76 t + t2  3025
t
(152  2t) (t) + 2.55t a b
55 c3 c  a11385186 tt2 b d
2 2
c2 
2.55t + t (152  2t)
7a 76 tt2 3025 b d  0
(152  2t) (t) + 2.55t a b
t 55
2 2 t2  109 t + 1298  0
c1  55 
2.55t + t(152  2t) 109 11092 4 (1298)
t t  13.61 mm ;
1 11385  186 t + t 2 2
c1 
2  131 + t

Problem 5.6-17 Determine the ratios of the weights of three beams that
have the same length, are made of the same material, are subjected to the
same maximum bending moment, and have the same maximum bending h = 2b a
stress if their cross sections are (1) a rectangle with height equal to twice
the width, (2) a square, and (3) a circle (see figures).
b a d

Solution 5.6-17 Ratio of weights of three beams


Beam 1: Rectangle (h  2b) a3
Beam 2: Square (a  side dimension) (2) SQUARE: S  a  (6S)1/3
6
Beam 3: Circle (d  diameter)
L, g, Mmax, and smax are the same in all three beams. A2  a2  (6S)2/3  3.3019 S2/3
M
S  section modulus S 
pd 3 32S 1/3
d a b
s
Since M and s are the same, the section moduli must (3) CIRCLE: S 
32 p
be the same.
pd 2 p 32S 2/3
(1) RECTANGLE: S 
bh2

2b3 3S 1/3
b a b A3   a b  3.6905 S2/3
6 3 2 4 4 p

3S 2/3 Weights are proportional to the cross-sectional areas


A1  2b2  2 a b  2.6207S2/3 (since L and g are the same in all 3 cases).
2
W1 : W2 : W3  A1 : A2 : A3
A1 : A2 : A3  2.6207 : 3.3019 : 3.6905
W1 : W2 : W3  1 : 1.260 : 1.408 ;
05Ch05.qxd 9/24/08 4:59 AM Page 425

SECTION 5.6 Design of Beams 425

Problem 5.6-18 A horizontal shelf AD of length L  915 mm, width t


b  305 mm, and thickness t  22 mm is supported by brackets at B and C
[see part (a) of the figure]. The brackets are adjustable and may be placed A D
in any desired positions between the ends of the shelf. A uniform load of B C
intensity q, which includes the weight of the shelf itself, acts on the shelf b
L
[see part (b) of the figure].
Determine the maximum permissible value of the load q if the allowable (a)
bending stress in the shelf is sallow  7.5 MPa and the position of the supports is q
adjusted for maximum load-carrying capacity.
A D
B C

L
(b)

Solution 5.6-18
NUMERICAL DATA Substitute x into the equation for either M1 or |M2|:
L  915 mm b  305 mm t  22 mm qL2
Mmax  (3  212) Eq. (1)
sallow  7.5 MPa 8
bt 2
Mmax  sallow S  sallow a b
MOMENT DIAGRAM
Eq. (2)
6
Equate Mmax from Eqs. (1) and (2) and solve for q:
4bt2sallow
qmax 
3L2(3  212)
Substitute numerical values:

For maximum load-carrying capacity, place the qmax  10.28 kN/m ;


supports so that M1  |M2|.
Let x  length of overhang
qL qx2
M1  (L  4x) |M2| 
8 2
qL qx2
‹ (L  4x) 
8 2
L
Solve for x: x  (12  1)
2
05Ch05.qxd 9/24/08 4:59 AM Page 426

426 CHAPTER 5 Stresses in Beams (Basic Topics)

Problem 5.6-19 A steel plate (called a cover plate) having cross-sectional


dimensions 6.0 in. * 0.5 in. is welded along the full length of the bottom flange
of a W 12 * 50 wide-flange beam (see figure, which shows the beam cross W 12  50
section).
What is the percent increase in the smaller section modulus (as compared to
the wide-flange beam alone)?
6.0  0.5 in. cover plate

Solution 5.6-19
NUMERICAL PROPERTIES FOR W 12 * 50 FIND I ABOUT HORIZ. CENTROIDAL AXIS
(FROM TABEL E-1(a)) d 2
Ih  I + A ac1  b +
1
(6) (0.5)3
A  14.6 in. 2
d  12.2 in. 2 12
0.5 2
+ (6) (0.5) a c2  b
d
c1  c2 c1 
2 2
I  391 in.4 S  64.2 in.3 Ih  491.411in.4
FIND CENTROID OF BEAM WITH COVER PLATE (TAKE FIND SMALLER SECTION MODULUS
1ST MOMENTS ABOUT TOP TO FIND c1 7 c2) Ih
Stop  Stop  68.419 in.3
c1
+ (6) (0.5) ad + b
d 0.5
A % increase in smaller section modulus
2 2
c1  c1  7.182 in. Stop  S
A + (6) (0.5) (100)  6.57% ;
S
c2  (d + 0.5)  c1 c2  5.518 in.

Problem 5.6-20 A steel beam ABC is simply supported 1.5 q


at A and B and has an overhang BC of length L  150 mm q
(see figure). The beam supports a uniform load of intensity
q  4.0 kN/m over its entire span AB and 1.5q over BC. A C
The cross section of the beam is rectangular with width b B 2b
and height 2b. The allowable bending stress in the steel is
sallow  60 MPa, and its weight density is   77.0 kN/m3. b
2L L
(a) Disregarding the weight of the beam, calculate the required width b of the rectangular cross section.
(b) Taking into account the weight of the beam, calculate the required width b.
05Ch05.qxd 9/24/08 4:59 AM Page 427

SECTION 5.6 Design of Beams 427

Solution 5.6-20
NUMERICAL DATA (b) NOW MODIFY-INCLUDE BEAM WEIGHT

L  150 mm q4
kN w  gA w  g 12b22
m
L2
kN Mmax  (1.5q + w)
sa  60 MPa g  77 3 2
m
 s a a b3 b
2
and Mmax
3
(a) IGNORE BEAM SELF WEIGHT-FIND bmin
L2 Equate Mmax1 to Mmax2 & solve for bmin
Mmax1  1.5 q at B
a sa b b3  1gL22 b2  qL2  0
2 2 3
2 3 4
and Mmax2  s a S S  b3
3
Insert numerical values, then solve for b
Equate Mmax1 to Mmax2 & solve for bmin bmin  11.92 mm ;
1
9 qL2 3
bmin a b
8 sa
bmin  11.91 mm ;

Problem 5.6-21 A retaining wall 5 ft high is constructed of 3 in.


horizontal wood planks 3 in. thick (actual dimension) that are p1 = 100 lb/ft2
supported by vertical wood piles of 12 in. diameter (actual 12 in.
dimension), as shown in the figure. The lateral earth pressure diam. 12 in.
is p1  100 lb/ft2 at the top of the wall and p2  400 lb/ft2 at diam.
the bottom.
Assuming that the allowable stress in the wood is 1200 psi,
calculate the maximum permissible spacing s of the piles. s
(Hint: Observe that the spacing of the piles may be 5 ft 3 in.
governed by the load-carrying capacity of either the planks
or the piles. Consider the piles to act as cantilever beams
subjected to a trapezoidal distribution of load, and consider
the planks to act as simple beams between the piles. To be on
the safe side, assume that the pressure on the bottom plank is Top view
uniform and equal to the maximum pressure.)
p2 = 400 lb/ft2
Side view
05Ch05.qxd 9/24/08 4:59 AM Page 428

428 CHAPTER 5 Stresses in Beams (Basic Topics)

Solution 5.6-21 Retaining wall

(1) PLANK AT THE BOTTOM OF THE DAM


t  thickness of plank  3 in.
b  width of plank (perpendicular to the plane
of the figure) q1  p1s
p2  maximum soil pressure q2  p2s
 400 lb/ft2  2.778 lb/in.2
d  diameter of pile  12 in.
s  spacing of piles
Divide the trapezoidal load into two triangles
q  p2b sallow  1200 psi
(see dashed line).
S  section modulus
sh2
(q1) (h)a b  (q2)(h)a b 
1 2h 1 h
qs2
p2bs 2
bt 2 Mmax  (2p1 p2)
Mmax   S 2 3 2 3 6
8 8 6
pd 3
p2bs 2 bt 2 S Mmax  sallow S or
Mmax  s allow S or  sallow a b 32
8 6
sh 2 pd 3
Solve for s: (2p1 + p2)  sallow a b
6 32
4sallow t 2 Solve for s:
s  72.0 in.
A 3p 2 3psallow d 3
s  81.4 in.
(2) VERTICAL PILE 16h2 (2p1 + p2)
h  5 ft  60 in.
PLANK GOVERNS smax  72.0 in. ;
p1  soil pressure at the top
 100 lb/ft 2  0.6944 lb/in.2

Problem 5.6-22 A beam of square cross section (a  length of each side) is bent in y
the plane of a diagonal (see figure). By removing a small amount of material at the top
and bottom corners, as shown by the shaded triangles in the figure, we can increase the
section modulus and obtain a stronger beam, even though the area of the cross section
is reduced. a ba

(a) Determine the ratio b defining the areas that should be removed in order to obtain
the strongest cross section in bending. z C
(b) By what percent is the section modulus increased when the areas are removed?
a ba
05Ch05.qxd 9/24/08 4:59 AM Page 429

SECTION 5.6 Design of Beams 429

Solution 5.6-22 Beam of square cross section with corners


removed
RATIO OF SECTION MODULI
S
 (1 + 3b)(1  b)2 Eq. (1)
S0

GRAPH OF EQ. (1)

a  length of each side


ba  amount removed
Beam is bent about the z axis.

ENTIRE CROSS SECTION (AREA 0)


a4 a I0 a3 12
I0  c0  S0   (a) VALUE OF b FOR A MAXIMUM VALUE OF S/S0
12 12 c0 12
a b 0
d S
SQUARE mnpq (AREA 1) db S0
(1  b)4a4 Take the derivative and solve this equation for b .
I1 
12 1
b ;
9
PARALLELOGRAM mm, n, n (AREA 2)
1 (b) MAXIMUM VALUE OF S/S0
I2  (base)(height)3
3
Substitute b  1/9 into Eq. (1). (S/S0)max  1.0535
(1  b)a 3 ba4
d 
1 The section modulus is increased by 5.35% when
I2  (ba 12)c (1  b)3 the triangular areas are removed. ;
3 12 6

REDUCED CROSS SECTION (AREA qmm, n, p, pq)


a4
I  I1 + 2I2  (1 + 3b)(1  b)3
12
(1  b) a I 12 a3
c S  (1 + 3b)(1  b)2
12 c 12

b

Problem 5.6-23 The cross section of a rectangular beam having 9
width b and height h is shown in part (a) of the figure. For reasons d
unknown to the beam designer, it is planned to add structural projections
of width b/9 and height d to the top and bottom of the beam [see part
(b) of the figure]. h h
For what values of d is the bending-moment capacity of the beam
increased? For what values is it decreased?
b d
b

9
(a) (b)
05Ch05.qxd 9/24/08 4:59 AM Page 430

430 CHAPTER 5 Stresses in Beams (Basic Topics)

Solution 5.6-23 Beam with projections


S2 d
Graph of versus
S1 h
d S2
h S1
0 1.000
0.25 0.8426
0.50 0.8889
0.75 1.0500
(1) ORIGINAL BEAM 1.00 1.2963

bh3 h I1 bh2
I1  c1  S1  
12 2 c1 6

(2) BEAM WITH PROJECTIONS

a bh + a b (h + 2d)3
1 8b 3 1 b
I2 
12 9 12 9
b
 [8h3 + (h + 2d)3]
108
h 1
c2  + d  (h + 2d) Moment capacity is increased when
2 2
d
I2 b[8h3 + (h + 2d)3] 7 0.6861 ;
S2   h
c2 54(h + 2d) Moment capacity is decreased when
d
RATIO OF SECTION MODULI 6 0.6861 ;
h
2d 3
3 3 8 + a1 + b NOTES:
S2 b [8h + (h + 2d) ] h
  2d 3
 1 when a1 + b  9a1 + b + 80
S2 2d
9(h + 2d)(bh2)
9 a1 + b
S1 2d
S1 h h
h
d
or  0.6861 and 0
EQUAL SECTION MODULI h
S2 d S2 d 14
3  1
Set  1 and solve numerically for . is minimum when   0.2937
S1 h S1 h 2

a b  0.8399
d d S2
 0.6861 and 0
h h S1 min
05Ch05.qxd 9/24/08 4:59 AM Page 431

SECTION 5.7 Nonprismatic Beams 431

Nonprismatic Beams
Problem 5.7-1 A tapered cantilever beam AB of length q = P/L
L has square cross sections and supports a concentrated load
P at the free end [see figure part (a)]. The width and height of B
the beam vary linearly from hA at the free end to hB at the A
hA
fixed end. A B
Determine the distance x from the free end A to the cross hB
section of maximum bending stress if hB  3hA. x
P Sliding
(a) What is the magnitude smax of the maximum bending x
stress? What is the ratio of the maximum stress to the L support
L
largest stress B at the support?
(b) Repeat (a) if load P is now applied as a uniform load (a)
(b)
of intensity q  P/L over the entire beam, A is restrained
by a roller support and B is a sliding support [see figure,
part (b)].

Solution 5.7-1
(a) FIND MAX. BENDING STRESS FOR TAPERED 2PL
sB  s(L) sB 
CANTILEVER
9hA 3
h(x)3
h(x)  hA a1 + b
2x
S(x)  4PL
L 6
smax 9hA 3 smax
M(x) 6(P)(x)  2 ;
s(x)  s(x)  sB 2PL sB
3
chA a 1 + bd
S(x) 2x 9hA 3
L
6PxL3 (b) REPEAT (A) BUT NOW FOR DISTRIBUTED UNIFORM
s(x) 
hA3 (L + 2x)3 LOAD OF P/L OVER ENTIRE BEAM

d
s(x)  0 then solve for xmax a Fv  0 RA  P
dx

M(x)  c c RA x  xa b d d
6PxL3 P x
c 3 d 0
d
dx hA (L + 2x)3 L 2

L3 L3 1 2P
c6P 36Px d 0 M(x)  Px  x
2 L
hA3 (L2x)3 hA3 (L2x)4
 L + 4x L 1 2P
0 so x Px  x
M(x) 2 L
hA3 (L + 2x)4 4 s(x)  s(x) 
2x 3
c hA a1 + bd
S(x)
L
6P L3 L
smax  sa b
L 4
smax  6
L 3
hA3 aL + 2 b
4
L2
4 s(x)  3xP (2L + x)
hA3 (L + 2x)3
4PL
smax  ;
9hA 3
05Ch05.qxd 9/24/08 4:59 AM Page 432

432 CHAPTER 5 Stresses in Beams (Basic Topics)

d xmax  0.20871 L
s(x)  0 then solve for xmax
dx smax  s (0.20871 L)
2
c 3xP (2Lx) 3 d 0
d L PL
smax  0.394 3 ;
dx hA (L2x)3 hA
L2
c 3P (2L + x)
sB  s(L) So
hA3(L + 2x) 3
a 0.39385 b
PL
L2 PL smax hA3
 3xP sB  
hA3 (L + 2x)3 9hA3 sB PL
2
d 0
L 9hA3
+ 18xP (2L + x)
hA3 (L + 2x) 4
smax
 3.54 ;
Simplifying sB
L2  5xL + x 2  0 so
xmax 5  152  4

L 2

Problem 5.7-2 A tall signboard is supported by two vertical beams consisting of thin-walled, tapered circular tubes
[see figure]. For purposes of this analysis, each beam may be represented as a cantilever AB of length L  8.0 m subjected to
a lateral load P  2.4 kN at the free end. The tubes have constant thickness t  10.0 mm and average diameters dA  90 mm
and dB  270 mm at ends A and B, respectively.
Because the thickness is small compared to the diameters, the moment of inertia at any cross section may be obtained
from the formula I  pd3t/8 (see Case 22, Appendix D), and therefore, the section modulus may be obtained from the formula
S  pd2t/4.
(a) At what distance x from the free end does the maximum bending stress occur? What is the magnitude smax of the
maximum bending stress? What is the ratio of the maximum stress to the largest stress sB at the support?
(b) Repeat (a) if concentrated load P is applied upward at A and downward uniform load q(x)  2P/L is applied over the
entire beam as shown. What is the ratio of the maximum stress to the stress at the location of maximum moment?

2P
q(x) = —
L
P = 2.4 kN
B
Wind A t
B
load A
x
P
d
L = 8.0 m
t = 10.0 mm x

L = 8.0 m

(b)
dA = 90 mm dB = 270 mm

(a)
05Ch05.qxd 9/24/08 4:59 AM Page 433

SECTION 5.7 Nonprismatic Beams 433

Solution 5.7-2
(a) FIND MAX. BENDING STRESS FOR TAPERED CANTILEVER PL
2 smax 2ptdA2
d(x)  d A a1 + b
2x pd(x) t
S(x)  
a b
L 4 sB 4 P L
P  2.4 kN 9 pt dA2
smax 9
L  8 m t  10 mm  ;
sB 8
dA  90 mm
Evaluate using numerical data
dB  270 mm (2400) (8)
smax 
M(x) 4P x 2p (0.010) (0.090)2
s(x)  s(x) 

J K
S(x) pt 2
c dA a 1 + bd
2x smax  37.7 MPa ;
L
(b) REPEAT (A) BUT NOW ADD DISTRIBUTED LOAD
xL2
c 2 d
4P
s(x) 
M(x)  aPx  2 x b
P x
pt dA (L + 2x)2
L 2
d
s(x)  0 then solve for xmax L + x
dx M(x)   Pxa b
L
xL2
c c 2 dd 0
d 4P L + x
dx pt dA (L + 2x)2  Px a b
M(x) L
L2 s(x)  s(x) 
c4
P 2x 2
cdA a1 + bd
S(x) pt
pt dA (L + 2x)2
2
4 L
xL2
d 0
P
 16 s(x)  4Px (Lx)
L
pt dA2 (L + 2x)3
ptdA2 (L2x)2
 L + 2x
or c4PL2 d 0 tension on top, compression on bottom of beam
ptdA2 (L + 2x)3
d
L s(x)  0 then solve for xmax
so xmax  4m ; dx
2
c 4Px (Lx) d 0
d L
 sa b
L
smax dx ptdA (L2x)2
2
2
c 4P ( L + x)
L
L 2
L ptdA2 (L + 2x)2
≥ ¥
4P 2
smax  L
L 2  4Px
aL + 2 b
pt
dA2 ptdA2 (L + 2x)2
2
d 0
L
PL 16Px (Lx)
smax  ptdA2 (L2x)3
2ptdA2
 L + 4x
Stress at support sB  s(L) OR simplifying c 4PL2 d 0
ptdA2 (L + 2x)3
4 P L L
sB  so xmax 
9 pt dA2 4
xmax  2 m ;
05Ch05.qxd 9/24/08 4:59 AM Page 434

434 CHAPTER 5 Stresses in Beams (Basic Topics)

stress at support
smax  s a b
L
4 sB  s(L)
L
sB  4PL ( L + L)
 4P aL b
L L L
ptdA2 (L + 2L2)
J b K
smax
L 2
p t dA2 aL2
4 4
sB  0 so no ratio of smax/sB is possible
4
PL MAX. MOMENT AT L/2 SO COMPARE
smax 
3 p t dA2 Stress at location of max. moment
evaluate using numerical data
sa b  4P aL b
L L L L
P  2.4 kN L8m
L 2
ptdA2 aL2 b
2 2 2
t  10 mm d A  90 mm
2
dB  270 mm
sa b  P
L 1 L
(2400) (8)
smax  2 4 ptdA2
3p (0.010) (0.090)2
smax  25.2 MPa ; PL
3ptdA2 4
smax/s(L/2)   ;
a P b
1 L 3
4 ptdA2

Problem 5.7-3 A tapered cantilever beam AB having rectangular cross sections is subjected to a concentrated
load P  50 lb and a couple M0  800 lb-in. acting at the free end [see figure part (a)]. The width b of the beam is
constant and equal to 1.0 in., but the height varies linearly from hA  2.0 in. at the loaded end to hB  3.0 in.
at the support.
(a) At what distance x from the free end does the maximum bending stress smax occur? What is the magnitude
smax of the maximum bending stress? What is the ratio of the maximum stress to the largest stress sB at
the support?
(b) Repeat (a) if, in addition to P and M0, a triangular distributed load with peak intensity q0  3P/L acts upward
over the entire beam as shown. What is the ratio of the maximum stress to the stress at the location of maximum
moment?

P = 50 lb 3P
P = 50 lb q0 = —
L

A M0 = 800 lb-in. B A M0 = 800 lb-in. B


hA = hB =
2.0 in. 3.0 in.

x x
b = 1.0 in. b = 1.0 in.
L = 20 in. L = 20 in.

(a) (b)
05Ch05.qxd 9/24/08 4:59 AM Page 435

SECTION 5.7 Nonprismatic Beams 435

Solution 5.7-3
(a) FIND MAX. BENDING STRESS FOR TAPERED CANTILEVER Px + M0
FIG. (A) s(x)  2
b c hA a1 + bd
x
h(x)  hA a1 + b
x
2L
2L
6
numerical data
L2
P  50 lb L  20 in. s(x)  24 1Px + M02
bhA2 (2L + x)2
hA  2 in. hB  3 in. b  1 in.
d
4 s(x)  0 then solve for xmax
M0  PL M0  800 in.-lb dx
5
L2
c24 1Px + M02 d 0
d
bh(x)3 I(x)
I(x)  S(x)  dx bhA2 (2L + x)2
12 h(x)
L2
2 24P
bh(x)2 bhA2 (2L + x)2
S(x)  L2
6  2 124Px 24M02 0
2 bhA2 (2Lx)3
b chA a1 + bd
x
2PL + Px + 2M0
OR simplifying c24L2 d 0
2L
S(x) 
6 bhA2 (2L + x)3
M(x)  Px + M0 2 1PL  M02
so x 
M(x) P
s(x)  xmax  8 in. ;
S(x)
agrees with plot at left

2000 Evaluate max. stress & stress at B using


numerical data
smax  s(8) smax  1250 psi ;
1500
M(x) sB  s(20) sB  1200 psi
(in.-lb) smax
1000  1.042 ;
sB
(b) FIND MAX. BENDING STRESS FOR TAPERED
500 CANTILEVER, FIG. (B)
0 10 20
h(x)  hA a1 + b
x (in.) x
2L
1260
4
M0  PL M0  800 in.-lb
5
1240 P
σ (x) q0  3
L
(psi)
1220 bh(x)3 I(x) bh(x)2
I(x)  S(x)  S(x) 
12 h(x) 6
2
1200
0 10 20
x (in.)
05Ch05.qxd 9/24/08 4:59 AM Page 436

436 CHAPTER 5 Stresses in Beams (Basic Topics)

2 d
b chA a1 + bd
x
s(x)  0 then solve for xmax
2L dx
S(x) 
s(x)  c124PL  12x2 q02
6 d
dx
1 x
a q0 b x
x
M(x)  Px + M0 + L
2 L 3  2 (24PxL + 24M0L
bhA2 (2L + x)2
M(x)
s(x) 
d 0
L
S(x)  4x3q0) *
bhA2 (2L + x)3
1500 Simplifying
12PL2 + 6PxL + 6x2 q0L
+ x3 q0 + 12M0 L  0
M(x)
1000
(in.-lb) Solve for xmax
xmax  4.642 in. ;
Max. stress & stress at B
500
0 10 20 smax  s (xmax)
x (in.)
smax  1235 psi ;
1400 sB  s (20) sB  867 psi
FIND MAX. MOMENT AND STRESS AT LOCATION OF MAX.
1200 MOMENT
σ (x)
q0x3
aPx + M0  b 0
(psi) d d
M(x)  0
1000 dx dx 6L
P (2L)
xm  xm  16.33 in.
800 A q0
0 10 20
x (in.) sm  s(xm) sm  1017 psi
smax
q0 x3  1.215 ;
Px + M0  sm
6L
s(x)  2
b chA a 1 + bd
x
2L
6
s(x)  416PxL  6 M0 L + x3 q02
L
*
bhA2 (2L + x)2
05Ch05.qxd 9/24/08 4:59 AM Page 437

SECTION 5.7 Nonprismatic Beams 437

Problem 5.7-4 The spokes in a large flywheel are modeled as beams fixed at one end and loaded by a force P and a
couple M0 at the other (see figure). The cross sections of the spokes are elliptical with major and minor axes (height and width,
respectively) having the lengths shown in the figure part (a). The cross-sectional dimensions vary linearly from end A to end B.
Considering only the effects of bending due to the loads P and M0, determine the following quantities.
(a) The largest bending stress sA at end A
(b) The largest bending stress sB at end B
(c) The distance x to the cross section of maximum bending stress
(d) The magnitude smax of the maximum bending stress
(e) Repeat (d) if uniform load q(x)  10P/3L is added to loadings P and M0, as shown in the figure part (b).

P = 12 kN 10P
q(x) = —
M0 = 10 kN•m B 3L

A
P
x
L = 1.25 m M0
A B
x
L = 1.25 m
hA = 90 mm hB = 120 mm
(b)

bA = 60 mm
bB = 80 mm

(a)

Solution 5.7-4
(a-d) FIND MAX. BENDING STRESS FOR TAPERED 30
CANTILEVER

numerical data
L  1.25 m bA  60 mm hA  90 mm M(x)
20
(kN•m)
bB  80 mm hB  120 mm
P  12 kn M0  10 kN # m

h(x)  hA a1 b b(x)  bA a1 + b


x x 10
3L 3L 0 0.5 1
x (m)
p b(x) h(x)3 I(x)
I(x)  S(x)  240
64 h(x)
2 230
p b(x) h(x)2
S(x)  σ (x)
32 220
(MPa)
x 3
p bA hA2 a1 + b 210
3L
S(x) 
32
200
0 0.5 1
x (m)
05Ch05.qxd 9/24/08 4:59 AM Page 438

438 CHAPTER 5 Stresses in Beams (Basic Topics)

M(x) p b(x) h(x)3 I(x)


M(x)  Px + M0 s(x)  I(x)  S(x) 
S(x) 64 h(x)
Px + M0 2
s(x) 
x 3 p b(x) h(x)2
p bA hA2 a1 + b S(x) 
3L 32
x 3
p bA hA2 a1 + b
32
L3 3L
s(x)  864 a b a b
Px + M0
S(x) 
p bA hA2 (3L + x)3 32
d 10 P x2
s (x)  0 then solve for xmax M(x)  P x + M0 
dx 3 L 2
M(x)
L3
c864 d 0
d Px + M0 s(x) 
S(x)
dx p bAhA (3L + x)3
2

15
P L3
864
p bAhA2 (3L + x)3
Px + M0 L3 10
 2592 0 M(x)
p bAhA (3L + x)4
2
(kN•m)
OR simplfying 5

3PL + 2Px + 3M0


c864L3 d 0
p bAhA2 (3L + x)4 0
0 0.5 1
x (m)
3(PL  M0)
so xmax 
2P 300
xmax  0.625 m ;
agrees with plot above 200
σ (x)
Evaluate using numerical data (MPa)
smax  s(xmax) smax  231 MPa ; 100
sA  s(0) sA  210 MPa ;
sB  s(L) sB  221 MPa ; 0
0 0.5 1
smax
 1.045 x (m)
sB
10 P x2
P x + M0 
(e) FIND MAX. BENDING STRESS INCLUDING 3 L 2
UNIFORM LOAD
s(x) 
x 3
p bA hA2 a1 + b
L  1.25 m bA  60 mm hA  90 mm
≥ ¥
3L
bB  80 mm hB  120 mm 32

s(x)  288 13 P x L  3 M0 L


P  12 kN M0  10 kN # m

h(x)  hA a1 + b
x L2
3L + 5 P x22
p bA hA2 (3 L + x)3
b(x)  bA a1 + b
x d
3L s(x)  0 then solve for xmax
dx
05Ch05.qxd 9/24/08 4:59 AM Page 439

SECTION 5.7 Nonprismatic Beams 439

OR
c 288 13 Px L  3 M0 L + 5 P x22
d
dx 9PL2  36PxL + 5Px2  9M0L  0
2
d 0
L Solving for x max: xmax  0.105m
*
pbAhA2 (3L + x)3
solution agrees with plot above, evaluate using
L2
s (x)  c(864 PL  2880 P x)
d
numerical data
dx p bA hA2 (3 Lx)3
smax  s(xmax) smax  214 MPa ;
3 1864 P x L864 M0 L1440Px22
sA  s(0) sA  210 MPa ;
L2
d 0
sB  s(L) sB  0 MPa ;
*
pbAhA2 (3L + x)4
OR simplifying

c9PL2  36PxL + 5Px2  9M0L d


(288 L2) 0
cpbAhA2 (3L + x)4 d

Problem 5.7-5 Refer to the tapered cantilever beam of solid circular cross section shown in Fig. 5-24 of Example 5-9.
(a) Considering only the bending stresses due to the load P, determine the range of values of the ratio dB/dA for which
the maximum normal stress occurs at the support.
(b) What is the maximum stress for this range of values?

Solution 5.7-5 Tapered cantilever beam

FROM EQ. (5-32), EXAMPLE 5-9 x 2 1


 [32Px][p] [3]cdA + (dB  dA)a b d c (dB  dA) d
32Px L L
s1  Eq. (1)
x 3
pcdA + (dB  dA)a b d After simplification:
L
x 2
N  32pPcdA + (dB  dA)a b d cdA  2(dB  dA) d
x
FIND THE VALUE OF x THAT MAKES s1 A MAXIMUM L L
x 6
va
du
b  ua b
dv D  p 2 cdA + (dB  dA) d
u ds1 dx dx N L
Let s1   
32PcdA  2(dB  dA) d
v dx v 2 D x
ds1 N L
x 3
N  pcdA + (dB  dA)a b d [32P]  
x 4
p cdA + (dB  dA)a b d
L dx D
L
05Ch05.qxd 9/24/08 4:59 AM Page 440

440 CHAPTER 5 Stresses in Beams (Basic Topics)

Maximum bending stress occurs at the support when


dA  2(dB  dA)a b  0
ds1 x
0
dx L dB
1 … … 1.5 ;
x dA 1 dA
‹   Eq. (2)
2(dB  dA)
2a  1b
L dB
(b) MAXIMUM STRESS (AT SUPPORT B)
dA Substitute x/L  1 into Eq. (1):
(a) GRAPH OF x/L VERSUS dB/dA (EQ. 2) 32PL
smax  ;
pdB3

Fully Stressed Beams


Problems 5.7-6 to 5.7-8 pertain to fully stressed beams of rectangular q
cross section. Consider only the bending stresses obtained from the
flexure formula and disregard the weights of the beams.
B
Problem 5.7-6 A cantilever beam AB having rectangular cross sections A
with constant width b and varying height hx is subjected to a uniform load hx hB
of intensity q (see figure).
How should the height hx vary as a function of x (measured from the x
free end of the beam) in order to have a fully stressed beam? (Express hx
in terms of the height hB at the fixed end of the beam.) L

hx hB

b
b

Solution 5.7-6 Fully stressed beam with constant width and varying height
hx  height at distance x AT THE FIXED END (x  L):
hB  height at end B
b  width (constant) 3q
hB  L
A bsallow
qx 2 bhx2 hx x hB x
AT DISTANCE x: M  S Therefore,  hx  ;
2 6 hB L L
2
M 3qx
sallow  
S bhx2
3q
hx  x
A bsallow
05Ch05.qxd 9/24/08 4:59 AM Page 441

SECTION 5.7 Fully Stressed Beams 441

Problem 5.7-7 A simple beam ABC having rectangular cross sections P


with constant height h and varying width bx supports a concentrated load h
P acting at the midpoint (see figure). A B C
How should the width bx vary as a function of x in order to have a
fully stressed beam? (Express bx in terms of the width bB at the midpoint
of the beam.) x

L L
— —
2 2

h h

bx bB

Solution 5.7-7 Fully stressed beam with constant height and varying width
h  height of beam (constant) AT MIDPOINT B (x  L/2)
bx  width at distance x from end Aa0 … x … b
L
3PL
2 bB 
bB  width at midpoint B (x  L/2) 2sallowh2
bx 2x 2bB x
Px 1 Therefore,  and bx  ;
AT DISTANCE x M  S  bx h2 bb L L
2 6 L
NOTE: The equation is valid for 0 … x … and the
M 3Px 3Px 2
sallow   bx  beam is symmetrical about the midpoint.
S bx h2 sallow h2

q
Problem 5.7-8 A cantilever beam AB having rectangular cross sections
with varying width bx and varying height hx is subjected to a uniform
load of intensity q (see figure). If the width varies linearly with x
according to the equation bx  bB x/L, how should the height hx vary as B
a function of x in order to have a fully stressed beam? (Express hx in
terms of the height hB at the fixed end of the beam.) A hx hB

hx hB

bx
bB
05Ch05.qxd 9/25/08 2:29 PM Page 442

442 CHAPTER 5 Stresses in Beams (Basic Topics)

Solution 5.7-8 Fully stressed beam with varying width and varying height
hx  height at distance x 3qLx
hB  height at end B hx 
A bB sallow
bx  width at distance x
bB  width at end B
AT THE FIXED END (x  L)
bx  bB a b
x
3qL2
L hB 
A bB sallow
AT DISTANCE x
qx 2 bx h2x bB x hx x x
M S  (hx)2 Therefore,  hx  hB ;
2 6 6L hB A L AL
M 3qLx
sallow  
S bB h2x

Shear Stresses in Rectangular Beams


Problem 5.8-1 The shear stresses t in a rectangular beam are given by
Eq. (5-39):
V h2
t a  y21 b
2I 4
in which V is the shear force, I is the moment of inertia of the cross-sectional
area, h is the height of the beam, and y1 is the distance from the neutral axis to
the point where the shear stress is being determined (Fig. 5-30).
By integrating over the cross-sectional area, show that the resultant
of the shear stresses is equal to the shear force V.

Solution 5.8-1 Resultant of the shear stresses


V  shear force acting on the cross section
R  resultant of shear stresses t
h/2 h/2
V h2
R tbdy1  2 a  y21 bbdy1
Lh/2 L0 2I 4
h/2 2
a  y21 bdy1
12V h
 (b)
bh L0
3 4
12V 2h3
 3 a b V
h 24
bh3 ‹ R  V Q.E.D. ;
I
12
V h2
t a  y21 b
2I 4
05Ch05.qxd 9/25/08 2:29 PM Page 443

SECTION 5.8 Shear Stresses in Rectangular Beams 443

Problem 5.8-2 Calculate the maximum shear stress tmax 22.5 kN/m
and the maximum bending stress smax in a wood beam
(see figure) carrying a uniform load of 22.5 kN/m (which 300 mm
includes the weight of the beam) if the length is 1.95 m and the
cross section is rectangular with width 150 mm and height
300 mm, and the beam is (a) simply supported as in the figure 150 mm
part (a) and (b) has a sliding support at right as in the figure 1.95 m
part (b).
(a)

22.5 kN/m

1.95 m

(b)

Solution 5.8-2
kN M
q  22 b  150 mm smax  smax  4.65 MPa ;
m S
h  300 mm L  1.95 m
(b) MAXIMUM SHEAR STRESS
(a) MAXIMUM SHEAR STRESS V  qL
qL 3V
V A  bh tmax  tmax  1430 kPa ;
2 2A
3V MAXIMUM BENDING STRESS
tmax  tmax  715 kPa ;
2A qL2
M
MAXIMUM BENDING STRESS 2
qL2 bh2 M
M S smax  smax  18.59 MPa ;
8 6 S

Problem 5.8-3 Two wood beams, each of rectangular cross section


(3.0 in.  4.0 in., actual dimensions) are glued together to form a 4.0 in.
solid beam of dimensions 6.0 in.  4.0 in. (see figure). The beam
is simply supported with a span of 8 ft. M
What is the maximum moment Mmax that may be 6.0 in.
applied at the left support if the allowable shear stress in the
glued joint is 200 psi? (Include the effects of the beam’s
own weight, assuming that the wood weighs 35 lb/ft3.)
8 ft
05Ch05.qxd 9/25/08 2:29 PM Page 444

444 CHAPTER 5 Stresses in Beams (Basic Topics)

Solution 5.8-3
L  8 ft b  4 in. M qL
V +
h  6 in. t allow  200 psi Ab#h L 2

a + b
lb 3V 3 M qL
g  35 tmax  
ft3 2A 2A L 2
q  g A weight of beam per unit distance 2 AL qL2
M tmax 
3 2
1b
q  5.833 2 AL qL2
ft Mmax  tallow 
3 2
Maximum load Mmax
Mmax  25.4 k-ft ;

Problem 5.8-4 A cantilever beam of length L  2 m supports a P = 8.0 kN


load P  8.0 kN (see figure). The beam is made of wood with
200 mm
cross-sectional dimensions 120 mm * 200 mm.
Calculate the shear stresses due to the load P at points located
25 mm, 75 mm, and 100 mm from the top surface of the L=2m
beam. from these results, plot a graph showing the distribution of 120 mm
shear stresses from top to bottom of the beam.

Solution 5.8-4 Shear stresses in a cantilever beam


Distance from the y1 t t
top surface (mm) (mm) (MPa) (kPa)
0 100 0 0
25 75 0.219 219
50 50 0.375 375
2 75 25 0.469 469
a  y21 b
V h
Eq. (5-39): t 
2I 4 100 (N.A.) 0 0.500 500
V  P  8.0 kN  8,000 N
GRAPH OF SHEAR STRESS t
bh3
I  80 * 106 mm4
12
h  200 mm (y1  mm)
(200)2
c y21 d
8,000
t (t  N/mm2  MPa)
2(80 * 106) 4
t  50 * 106(10,000  y21) (y1  mm; t  MPa)
05Ch05.qxd 9/25/08 2:29 PM Page 445

SECTION 5.8 Shear Stresses in Rectangular Beams 445

Problem 5.8-5 A steel beam of length L  16 in. and cross- q = 240 lb/in.
sectional dimensions b  0.6 in. and h  2 in. (see figure) supports a
uniform load of intensity q  240 lb/in., which includes the weight
h = 2 in.
of the beam.
Calculate the shear stresses in the beam (at the cross
section of maximum shear force) at points located 1/4 in., 1/2 in., b = 0.6 in.
3/4 in., and 1 in. from the top surface of the beam. From these L = 16 in.
calculations, plot a graph showing the distribution of shear stresses
from top to bottom of the beam.

Solution 5.8-5 Shear stresses in a simple beam


Distance from the y1 t
top surface (in.) (in.) (psi)
0 1.00 0
0.25 0.75 1050
0.50 0.50 1800
0.75 0.25 2250
V h2
Eq. (5-39): t  a  y21 b 1.00 (N.A.) 0 2400
2I 4
qL bh3 GRAPH OF SHEAR STRESS t
V  1920 lb I   0.4 in.4
2 12

UNITS: POUNDS AND INCHES


1920 (2)2
t c  y21  (2400)(1  y21) d
2(0.4) 4
(t  psi; y1  in.)

Problem 5.8-6 A beam of rectangular cross section (width b and height h)


supports a uniformly distributed load along its entire length L. The
allowable stresses in bending and shear are sallow and tallow, respectively.
(a) If the beam is simply supported, what is the span length L0 below
which the shear stress governs the allowable load and above which the
bending stress governs?
(b) If the beam is supported as a cantilever, what is the length L0
below which the shear stress governs the allowable load and above which
the bending stress governs?
05Ch05.qxd 9/25/08 2:29 PM Page 446

446 CHAPTER 5 Stresses in Beams (Basic Topics)

Solution 5.8-6 Beam of rectangular cross section


b  width h  height L  length (b) CANTILEVER BEAM
Uniform load q  intensity of load BENDING
ALLOWABLE STRESSES sallow and tallow qL2 bh2
Mmax  S
(a) SIMPLE BEAM 2 6
Mmax 3qL2
BENDING smax  
S bh2
qL2 bh2
Mmax  S sallowbh2
8 6 qallow  (3)
3 3L2
Mmax 3qL
smax   SHEAR
S 4bh2
4sallow bh2 Vmax  qL A  bh
qallow  2
(1)
3L 3V 3qL
tmax  
SHEAR 2A 2bh

qL 2tallow bh
Vmax  A  bh qallow  (4)
2 3L

3V 3qL Equate (3) and (4) and solve for L0:


tmax  
L0  a b
2A 4bh h sallow
;
4tallowbh 2 tallow
qallow  (2)
3L
NOTE: If the actual length is less than L 0, the shear
Equate (1) and (2) and solve for L0: stress governs the design. If the length is greater than
L0, the bending stress governs.
b
sallow
L 0  ha ;
tallow

Problem 5.8-7 A laminated wood beam on simple supports is built up P 2 in.


by gluing together four 2 in.  4 in. boards (actual dimensions) to form 3 ft
a solid beam 4 in.  8 in. in cross section, as shown in the figure. 2 in.
The allowable shear stress in the glued joints is 65 psi, and the allowable 2 in.
bending stress in the wood is 1800 psi.
2 in.
If the beam is 9 ft long, what is the allowable load P acting at the
one-third point along the beam as shown? (Include the effects of the L  9 ft
beam’s own weight, assuming that the wood weighs 35 lb/ft3.) 4 in.

Solution 5.8-7
L  9 ft b  4 in. 1b
q  7.778
h  8 in. A  bh ft
t allow  65 psi s allow  1800 psi ALLOWABLE LOAD BASED UPON SHEAR STRESS IN THE
GLUED JOINTS; MAX. SHEAR STRESS AT NEUTRAL AXIS

WEIGHT OF BEAM PER UNIT DISTANCE VQ 3V


t tmax 
lb Ib 2A
g  35 q  gA
ft3
05Ch05.qxd 9/25/08 2:29 PM Page 447

SECTION 5.8 Shear Stresses in Rectangular Beams 447

2 qL ALLOWABLE LOAD BASED UPON BENDING STRESS


VP +
3 2 2 qL q
MP 3 ft + 3 ft  (3 ft)2
aP + b
3V 3 2 qL 3 2 2
tmax  
2A 2A 3 2 b h2
S
P  a A tmax  b
3 qL 6
4 2 qL q
P3 ft + 3 ft  (3 ft)2
3 qL M 3 2 2
Pmax  A t allow  smax  
4 S S

 a
Pmax  2.03 k (governs) sallow S3 3 qL q
Pmax   (3ft)b
(3 ft) 2 2 2 2
Pmax  3.165 k
P allow  2.03 k ;

Problem 5.8-8 A laminated plastic beam of square cross M


section is built up by gluing together three strips, each q
10 mm  30 mm in cross section (see figure). The beam has
a total weight of 3.6 N and is simply supported with span 10 mm
length L  360 mm. 10 mm 30 mm
Considering the weight of the beam (q) calculate the 10 mm
maximum permissible CCW moment M that may be placed
at the right support. L
30 mm
(a) If the allowable shear stress in the glued joints is
0.3 MPa.
(b) If the allowable bending stress in the plastic is 8 MPa.

Solution 5.8-8
(a) FIND M BASED ON ALLOWABLE SHEAR STRESS IN GLUED b h2
JOINT
bh h b h2 Q 9
b  30 mm h  30 mm ta  0.3 MPa Q Q  2 3
3 3 9 Ib b h
W  3.6 N L  360 mm 12
W Q 4
q 
L Ib 3bh

M  L cta a b  d
N Ib qL
q  10 beam distributed weight
m Q 2

M  L cta a b  d
MAX. SHEAR ST LEFT SUPPORT 3bh qL
4 2
and Vm  t a a b
qL M Ib
Vm  + Mmax  72.2 N # M ;
2 L Q
Vm Q 3 2 3
bh b h
ta  I Ib 
Ib 12 12
05Ch05.qxd 9/25/08 2:29 PM Page 448

448 CHAPTER 5 Stresses in Beams (Basic Topics)

(b) FIND M BASED ON ALLOWABLE BENDING STRESS AT


Mm  a + b a + b
qL M L M
h/2 FROM NA AT LOCATION (xm) OF MAX. BENDING 2 L 2 qL
MOMENT, Mm
M 2
qa b
L
qx2
M(x)  a + bx
qL M d +
M(x)  0 2 qL
2 L 2 dx 
2
use to find location of zero shear where
max. moment occurs simplifying

qx2 1 1qL + 2 M2
2 2
ca + bx d
d qL M
Mm 
dx 2 L 2 8q L2
1 M bh2
 qL +  qx  0 also Mm  sa S Mm  sa a b
2 L 6
L M Equating both Mm expressions & solving for M where
xm  +
2 qL sa  8 MPa
MAX. MOMENT Mm bh2
2 sa a b a8 qL2 b  qL2
A
Mm  a + b xm 
qL M qxm 6
M
2 L 2 2
Mmax  9.01 N # m ;

Problem 5.8-9 A wood beam AB on simple supports with span length 7500 lb
18,500 ft-lb
equal to 10 ft is subjected to a uniform load of intensity 125 lb/ft acting 3 ft
along the entire length of the beam, a concentrated load of magnitude 125 lb/ft
7500 lb acting at a point 3 ft from the right-hand support, and a moment at
A of 18,500 ft-lb (see figure). The allowable stresses in bending and shear, A B
respectively, are 2250 psi and 160 psi.
(a) From the table in Appendix F, select the lightest beam that will 10 ft
support the loads (disregard the weight of the beam).
(b) Taking into account the weight of the beam (weight density 5 35 lb/ft3),
verify that the selected beam is satisfactory, or if it is not, select a new beam.

Solution 5.8-9
1b RB  7.725 * 103 1b
(a) q  125 P  75001b M  18500 ft-b
ft Vmax  RB Vmax  7.725 * 103 1b
L  10 ft d  3 ft qd2
sAllow  2250 psi t allow  160 psi Mmax  RB d 
2
qL d M Mmax  2.261 * 104 1b-ft
RA  + P 
2 L L 3V 3Vmax
RA  1.025 * 103 1b tmax  Areq 
2A 2tallow
qL Ld M Areq  72.422 in.2
RB  + P +
2 L L
05Ch05.qxd 9/25/08 2:29 PM Page 449

SECTION 5.8 Shear Stresses in Rectangular Beams 449

M Mmax 3Vmax
smax  Sreq  Sreq  120.6 in.3 Vmax  RB Areq 
S sallow 2 tallow
From Appendix F: Select 8 * 12 in. beam (nominal Areq  73.405 in.2 < A
dimensions) ; 8 * 12 beam is still satisfactory for shear.
A  86.25 in.2 S  165.3 in.3 1b
qtotal  q + qbeam q total  145.964
ft
(b) REPEAT (A) CONSIDERING THE WEIGHT OF THE BEAM
qd2
1b Mmax  RB d 
g  35 qbeam  g A 2
ft3
Mmax  2.293 * 104 1b-ft
1b
q beam  20.964 Mmax
ft Sreq  Sreq  122.3 in.3 < S
sallow
qbeam L
RB  7.725 * 103 1b + 8 * 12 beam is still satisfactory for moment.
2
Use 8 * 12 in. beam ;
RB  7.83 * 103 1b

Problem 5.8-10 A simply supported wood beam of rectangular P


cross section and span length 1.2 m carries a concentrated load
P at midspan in addition to its own weight (see figure). The cross 240 mm
section has width 140 mm and height 240 mm. The weight density
of the wood is 5.4 kN/m3. 140 mm
Calculate the maximum permissible value of the load P if 0.6 m 0.6 m
(a) the allowable bending stress is 8.5 MPa, and (b) the allowable
shear stress is 0.8 MPa.

Solution 5.8-10 Simply supported wood beam


P (a) ALLOWABLE P BASED UPON BENDING STRESS
Mmax
240 mm sallow  8.5 MPa s 
S
140 mm PL qL2 P(1.2 m)
0.6 m 0.6 m Mmax  + 
4 8 4
b  140 mm h  240 mm (181.44 N/m)(1.2 m)2
+
A  bh  33,600 mm2 8
bh2  0.3 P + 32.66 N # m
S  1344 * 103 mm3
6 (P  newtons; M  N # m)
g  5.4 kN/m3 Mmax  Ssallow  (1344 * 103 mm3)(8.5 MPa)
L  1.2 m q  gbh  181.44 N/m  11,424 N # m
Equate values of Mmax and solve for P:
0.3P + 32.66  11,424 P  37,970 N
or P  38.0 kN ;
05Ch05.qxd 9/25/08 2:29 PM Page 450

450 CHAPTER 5 Stresses in Beams (Basic Topics)

(b) ALLOWABLE LOAD P BASED UPON SHEAR STRESS Equate values of V and solve for P:
3V P
tallow  0.8 MPa t  + 108.86  17,920 P  35,622 N
2A 2
P qL P (181.44 N/m)(1.2 m) or P  35.6 kN ;
V +  +
2 2 2 2 NOTE: The shear stress governs and
P Pallow  35.6 kN
 + 108.86 (N)
2
2At 2
V  (33,600 mm2)(0.8 MPa)  17,920 N
3 3

Problem 5.8-11 A square wood platform, 8 ft * 8 ft in area,


rests on masonry walls (see figure). The deck of the platform is
constructed of 2 in. nominal thickness tongue-and-groove planks 8 ft
(actual thickness 1.5 in.; see Appendix F) supported on two 8 ft
8-ft long beams. The beams have 4 in. * 6 in. nominal
dimensions (actual dimensions 3.5 in. * 5.5 in.).
The planks are designed to support a uniformly distributed
load w (lb/ft2) acting over the entire top surface of the platform.
The allowable bending stress for the planks is 2400 psi and
the allowable shear stress is 100 psi. When analyzing the
planks, disregard their weights and assume that their reactions
are uniformly distributed over the top surfaces of the supporting
beams.
(a) Determine the allowable platform load w1 (lb/ft2) based
upon the bending stress in the planks.
(b) Determine the allowable platform load w2 (lb/ft2) based
upon the shear stress in the planks.
(c) Which of the preceding values becomes the allowable
load wallow on the platform?
(Hints: Use care in constructing the loading diagram for the
planks, noting especially that the reactions are distributed loads
instead of concentrated loads. Also, note that the maximum shear
forces occur at the inside faces of the supporting beams.)
05Ch05.qxd 9/25/08 2:29 PM Page 451

SECTION 5.8 Shear Stresses in Rectangular Beams 451

Solution 5.8-11 Wood platform with a plank deck


Load on one plank:
w(lb/ft2)
q c d(b in.) 
wb
2 2
(lb/in.)
144 in. / ft 144

b a b(48) 
96 in. wb wb
Reaction R  qa
2 144 3
(R  lb; w  lb/ft2; b  in.)
Mmax occurs at midspan.
q(48 in.)2
b 
3.5 in. 89 in.
Mmax  Ra +
2 2 3
wb wb 89
 (46.25)  (1152)  wb
3 144 12
Platform: 8 ft * 8 ft (M  lb-in.; w  lb/ft2; b  in.)
t  thickness of planks Allowable bending moment:
 1.5 in. Mallow  s allow S  (2400 psi)(0.375 b)
w  uniform load on the deck (lb/ft ) 2
 900 b (lb-in.)
sallow  2400 psi Equate Mmax and Mallow and solve for w:
tallow  100 psi 89
wb  900 b w1  121 lb/ft2 ;
2
Find wallow (lb/ft ) 12

(a) ALLOWABLE LOAD BASED UPON BENDING STRESS IN THE (b) ALLOWABLE LOAD BASED UPON SHEAR STRESS IN THE

PLANKS PLANKS

Let b  width of one plank (in.) See the free-body diagram in part (a).

A  1.5b (in.2) Vmax occurs at the inside face of the support.

b  44.5q
89 in.
b Vmax  qa
S (1.5 in.)2 2
6
b 
wb 89 wb
 0.375b (in.3)  (44.5)a
144 288
Free-body diagram of one plank supported on the
(V  lb; w  lb/ft2; b  in.)
beams:
Allowable shear force:
3V 2Atallow
t Vallow 
2A 3
2(1.5 b)(100 psi)
  100 b (lb)
3
Equate Vmax and Vallow and solve for w:
89wb
 100b w2  324 lb/ft2 ;
288
(c) ALLOWABLE LOAD
Bending stress governs. wallow  121 lb/ft2 ;
05Ch05.qxd 9/25/08 2:29 PM Page 452

452 CHAPTER 5 Stresses in Beams (Basic Topics)

Problem 5.8-12 A wood beam ABC with simple supports 3P


PL
at A and B and an overhang BC has height h  300 mm L
— M = –––
2 2
(see figure). The length of the main span of the beam is
L  3.6 m and the length of the overhang is L/3  1.2 m. A h=
C
The beam supports a concentrated load 3P  18 kN at the B 300 mm
midpoint of the main span and a moment PL/2  10.8 kN . m
at the free end of the overhang. The wood has weight L b
density g  5.5 kN/m3. L —
3
(a) Determine the required width b of the beam
based upon an allowable bending stress of 8.2 MPa.
(b) Determine the required width based upon an allowable
shear stress of 0.7 MPa.

Solution 5.8-12
Numerical data: Mmax 6 Mmax
s 
L  3.6 m h  300 mm S bh2
PL 3PL
A  bh P  6 kN M b b  87.8 mm ;
2 sallow h2
kN
g  5.5 qbeam  g A (b) REQUIRED WIDTH b BASED UPON SHEAR STRESS
m3 tallow  0.7 MPa
Reactions, max. shear and moment equations 8
Vmax  2 P + q L
9 beam
3P M 4 4
RA   + qbeam L  P  qbeam L 3 Vmax 3 Vmax
2 L 9 9 t 
2A 2 bh
3P M 8 8
RB  + + qbeam L  2 P + qbeam L
a2 P + qbeam Lb 
2 L 9 9 3 8 3P 4
 + gL
2 bh 9 bh 3
8
Vmax  RB  2 P + q L
9 beam b
3P
b  89.074 mm
h a t allow
L L2 PL 17 4
MD  RA  qbeam   q L2  gLb
2 2 2 18 beam 3
Shear stress governs
PL
MB  b  89.1 mm ; (governs)
2

(a) REQUIRED WIDTH b BASED UPON BENDING STRESS


sallow  8.2 MPa
PL
Mmax  MB 
2
05Ch05.qxd 9/25/08 2:29 PM Page 453

SECTION 5.9 Shear Stresses in Circular Beams 453

Shear Stresses in Circular Beams


Problem 5.9-1 A wood pole of solid circular cross section (d  diameter) q0 = 20 lb/in.
is subjected to a horizontal force P  450 lb (see figure). The length of the pole is
L  6 ft, and the allowable stresses in the wood are 1900 psi in bending and
120 psi in shear.
d
Determine the minimum required diameter of the pole based upon
d
(a) the allowable bending stress, and (b) the allowable shear stress. L

Solution 5.9-1
1b 3
q  20 L  6 ft 32 Mmax
dmin 
in A p sallow
s allow  1900 psi
dmin  5.701 in.
t allow  120 psi
qL (b) BASED UPON SHEAR STRESS
Vmax  Vmax  720 1b
2 4V 16V
t 
qL 2 L 3A 3pd2
Mmax  Mmax  2.88 * 103 1b-ft
2 3 16 Vmax
dmin  dmin  3.192 in.
(a) BASED UPON BENDING STRESS A 3p tallow
M 32 M Bending stress governs dmin  5.70 in. ;
s 
S pd3

Problem 5.9-2 A simple log bridge in a remote area consists of two


parallel logs with planks across them (see figure). The logs
are Douglas fir with average diameter 300 mm. A truck moves
slowly across the bridge, which spans 2.5 m. Assume that the weight
of the truck is equally distributed between the two logs.
Because the wheelbase of the truck is greater than 2.5 m, only
one set of wheels is on the bridge at a time. Thus, the wheel load on
one log is equivalent to a concentrated load W acting at any position
along the span. In addition, the weight of one log and the planks it
supports is equivalent to a uniform load of 850 N/m acting on the log.
Determine the maximum permissible wheel load W based upon W
x
(a) an allowable bending stress of 7.0 MPa, and (b) an allowable
850 N/m
shear stress of 0.75 MPa.
300 mm

2.5 m
05Ch05.qxd 9/25/08 2:29 PM Page 454

454 CHAPTER 5 Stresses in Beams (Basic Topics)

Solution 5.9-2 Log bridge

Diameter d  300 mm (b) BASED UPON SHEAR STRESS


sallow  7.0 MPa Maximum shear force occurs when wheel is adja-
tallow  0.75 MPa cent to support (x  0).
Find allowable load W
qL 1
(a) BASED UPON BENDING STRESS Vmax  W +  W + (850 N/m)(2.5 m)
2 2
Maximum moment occurs when wheel is at midspan
(x  L/2).  W + 1062.5 N (W  newtons)
WL qL 2 pd2
Mmax  + A  0.070686 m2
4 8 4
4Vmax

W 1
(2.5 m) + (850 N/m)(2.5 m)2 tmax 
4 8 3A
3Atallow 3
 0.625W + 664.1 (N # m) (W  newtons) Vmax   (0.070686 m2)(0.75 MPa)
4 4
pd3
S  2.651 * 103m3  39,760 N
32
‹ W + 1062.5 N  39,760 N
Mmax  Ssallow  (2.651 * 103 m3)(7.0 MPa)
W  38,700 N  38.7 kN ;
 18,560 N # m
‹ 0.625W + 664.1  18,560
W  28,600 N  28.6 kN ;

Problem 5.9-3 A sign for an automobile service station is supported by b


two aluminum poles of hollow circular cross section, as shown in the
figure. The poles are being designed to resist a wind pressure of 75 lb/ft2
against the full area of the sign. The dimensions of the poles and sign d
Wind t=—
are h1  20 ft, h2  5 ft, and b  10 ft. To prevent buckling of the walls of h2
load
10
the poles, the thickness t is specified as one-tenth the outside diameter d.
(a) Determine the minimum required diameter of the poles based
upon an allowable bending stress of 7500 psi in the aluminum. d
(b) Determine the minimum required diameter based upon an
allowable shear stress of 2000 psi. h1

Probs. 5.9.3 and 5.9.4


05Ch05.qxd 9/25/08 2:29 PM Page 455

SECTION 5.9 Shear Stresses in Circular Beams 455

Solution 5.9-3 Wind load on a sign

b  width of sign (b) REQUIRED DIAMETER BASED UPON SHEAR STRESS


b  10 ft Vmax  W  1875 lb
p  75 lb/ft2
4V r22 + r2r1 + r12
a b
sallow  7500 psi d
t r2 
tallow  2000 psi 3A r2 2 + r1 2 2
d  diameter W  wind force on one pole d d d 2d
r1  t  
2 2 10 5
W  ph2 a b  1875 lb
d b
t
10 2 r22 + r2r1 + r21
(a) REQUIRED DIAMETER BASED UPON BENDING STRESS r2 2 + r1 2
d 2 2d 2
a b + a ba b + a b
d 2d
b  506,250 lb-in.
h2
Mmax  Wah1 + 2 2 5 5 61
2  
2 2
a b + a b
d 2d 41
p 4
I (d 4  d24) d2  d d1  d  2t  d 2 5
64 2 5
4d 2 9pd2
(d2  d21)  cd2  a b d 
p 2 p
4d 4 pd 4 369
I  cd 4  a b d  a b
p A
4 4 5 100
64 5 64 625
a ba b  7.0160 2
4V 61 100 V
369pd 4 t
 (in.4) 3 41 9pd2 d
40,000
7.0160 Vmax
d d2 
c (d  inches) tallow
2
(7.0160)(1875 lb)
Mc M(d/2) 17.253 M   6.5775 in.2
s   2000 psi
I 369pd 4/40,000 d3
d  2.56 in. ;
17.253 Mmax (17.253)(506,250 lb-in.)
d3   (Bending stress governs.)
sallow 7500 psi
 1164.6 in. 3
d  10.52 in. ;

Problem 5.9-4 Solve the preceding problem for a sign and poles
having the following dimensions: h1  6.0 m, h2  1.5 m, b  3.0 m,
and t  d/10. The design wind pressure is 3.6 kPa, and the allowable
stresses in the aluminum are 50 MPa in bending and 14 MPa in shear.
05Ch05.qxd 9/25/08 2:29 PM Page 456

456 CHAPTER 5 Stresses in Beams (Basic Topics)

Solution 5.9-4 Wind load on a sign

b  width of sign (b) REQUIRED DIAMETER BASED UPON SHEAR STRESS


b  3.0 m Vmax  W  8.1 kN
p  3.6 kPa 4V r2 2 + r1 r2 + r1 2
a b
d
t r2 
sallow  50 MPa 3A r2 2 + r1 2 2

tallow  16 MPa d d d 2d
r1  t  
2 2 10 5
d  diameter W  wind force on one pole
r2 2 + r1r2 + r1 2
W  ph2 a b  8.1 kN
d b
t r2 2 + r1 2
10 2
d 2 2d 2
a b + a ba b + a b
d 2d
(a) REQUIRED DIAMETER BASED UPON BENDING STRESS
2 5 5 5 61
 
b  54.675 kN # m
h2
d 2 2d 2
a b + a b
Mmax  Wah1 + 41
2
2 5
Mc p 4 p
s I (d  d41) A (d 2  d1 2)
I 64 2 4 2
4 4d 2 9pd2
cd  a b d 
d2  d d1  d  2t  d p 2
5 
4 5 100
4d 4
cd  a b d
p 4
a ba b  7.0160 2
I 4V 61 100 V
64 5 t
3 41 9pd2 d
pd 4 369 369pd 4
 a b (m 4) d2 
7.0160 Vmax

(7.0160)(8.1 kN)
64 625 40,000 tallow 14 MPa
d  0.004059 m2
c (d  meters)
2
d  0.06371 m  63.7 mm ;
Mc M(d/2) 17.253 M
s  4
 Bending stress governs
I 369pd /40,000 d3
17.253Mmax (17.253)(54.675 kN # m)
d3 
sallow 50 MPa
 0.018866 m3
d  0.266 m  266 m ;
05Ch05.qxd 9/25/08 2:29 PM Page 457

SECTION 5.10 Shear Stresses in Beams with Flanges 457

Shear Stresses in Beams with Flanges


Problem 5.10-1 through 5.10-6 A wide-flange beam (see figure) having y
the cross section described below is subjected to a shear force V. Using the
dimensions of the cross section, calculate the moment of inertia and then
determine the following quantites:
(a) The maximum shear stress tmax in the web.
(b) The minimum shear stress tmin in the web. z
O h1 h
(c) The average shear stress taver (obtained by dividing the shear t
force by the area of the web) and the ratio tmax/taver.
(d) The shear force Vweb carried in the web and the ratio Vweb /V.
NOTE: Disregard the fillets at the junctions of the web and flanges and b
determine all quantities, including the moment of inertia, by considering Probs 5.10.1through 5.-10.6
the cross section to consist of three rectangles.

Problem 5.10-1 Dimensions of cross section: b  6 in., t  0.5 in.,


h  12 in., h1  10.5 in., and V  30 k.

Solution 5.10-1 Wide-flange beam


(b) MINIMUM SHEAR STRESS IN THE WEB (Eq. 5-48b)
b  6.0 in.
Vb 2
t  0.5 in. tmin  (h  h12)  4555 psi ;
8It
h  12.0 in.
(c) AVERAGE SHEAR STREAR IN THE WEB (Eq. 5-50)
h1  10.5 in.
V
V  30 k taver   5714 psi ;
th1
tmax
MOMENT OF INERTIA (Eq.5-47)  1.014 ;
taver
1
I (bh3  bh31 + th31)  333.4 in.4 (d) SHEAR FORCE IN THE WEB (Eq. 5-49)
12
th1
(a) MAXIMUM SHEAR STRESS IN THE WEB (Eq. 5-48a) Vweb  (2tmax + tmin)  28.25 k ;
3
V
tmax  (bh2  bh21 + th21)  5795 psi ; Vweb
8It  0.942 ;
V
05Ch05.qxd 9/25/08 2:29 PM Page 458

458 CHAPTER 5 Stresses in Beams (Basic Topics)

Problem 5.10-2 Dimensions of cross section: b  180 mm, t  12 mm,


h  420 mm, h1  380 mm, and V  125 kN.

Solution 5.10-2 Wide-flange beam


b  180 mm (b) MINIMUM SHEAR STRESS IN THE WEB (Eq. 5-48b)
t  12 mm Vb 2
tmin  (h  h21)  21.86 MPa ;
h  420 mm 8It
(c) AVERAGE SHEAR STRESS IN THE WEB (Eq. 5-50)
h1  380 mm
V
V  125 kN taver   27.41 MPa ;
th1
tmax
MOMENT OF INERTIA (Eq. 5-47)  1.037 ;
taver
1
I (bh3  bh31 + th31)  343.1 * 106 mm4 (d) SHEAR FORCE IN THE WEB (Eq. 5-49)
12
th1
(a) MAXIMUM SHEAR STRESS IN THE WEB (Eq. 5-48a) Vweb  (2tmax + tmin)  119.7 kN ;
3
V
tmax  (bh2  bh21 + th21)  28.43 MPa ; Vweb
8It  0.957 ;
V

Problem 5.10-3 Wide-flange shape, W 8 * 28 (see Table E-1(a),


Appendix E); V  10 k.

Solution 5.10-3 Wide-flange beam

W 8 * 28 (b) MINIMUM SHEAR STRESS IN THE WEB (Eq. 5-48b)


b  6.535 in. Vb 2
tmin  (h  h21)  4202 psi ;
t  0.285 in. 8It

h  8.06 in. (c) AVERAGE SHEAR STRESS IN THE WEB (Eq. 5-50)
h1  7.13 in. taver 
V
 4921 psi ;
th1
V  10 k
tmax
MOMENT OF INERTIA (Eq. 5-47)  0.988 ;
taver
1
I (bh3  bh31 + th31)  96.36 in.4 (d) SHEAR FORCE IN THE WEB (EQ. 5-49)
12
th1
(a) MAXIMUM SHEAR STRESS IN THE WEB (Eq. 5-48a) Vweb  (2tmax + tmin)  9.432 k ;
3
V
tmax  (bh2  bh21 + th21)  4861 psi ; Vweb
8It  0.943 ;
V
05Ch05.qxd 9/25/08 2:29 PM Page 459

SECTION 5.10 Shear Stresses in Beams with Flanges 459

Problem 5.10-4 Dimensions of cross section: b  220 mm, t  12 mm,


h  600 mm, h1  570 mm, and V  200 kN .

Solution 5.10-4 Wide-flange beam


b  220 mm (c) AVERAGE SHEAR STRESS IN THE WEB (EQ. 5-50)
t  12 mm V
taver   29.24 MPa ;
h  600 mm th1

h1  570 mm tmax
 1.104
taver
V  200 kN
(d) SHEAR FORCE IN THE WEB (Eq. 5-49)
MOMENT OF INERTIA (Eq. 5-47) th1
Vweb  (2tmax + tmin)  196.1 kN ;
1 3
I (bh3  bh31 + th31)  750.0 * 106 mm4 Vweb
12  0.981 ;
V
(a) MAXIMUM SHEAR STRESS IN THE WEB (Eq. 5-48a)
V
tmax  (bh2  bh21 + th21)  32.28 MPa ;
8It
(b) MINIMUM SHEAR STRESS IN THE WEB (EQ. 5-48b)
Vb 2
tmin  (h  h21)  21.45 MPa ;
8It

Problem 5.10-5 Wide-flange shape, W 18 * 71


(see Table E-1(a), Appendix E); V  21 k.

Solution 5.10-5 Wide-flange beam


W 18 * 71 (b) MINIMUM SHEAR STRESS IN THE WEB (EQ. 5-48b)
b  7.635 in. Vb 2
tmin  (h  h21)  1993 psi ;
t  0.495 in. 8It

h  18.47 in. (c) AVERAGE SHEAR STRESS IN THE WEB (EQ. 5-50)
h1  16.85 in. taver 
V
 2518 psi ;
th1
V  21 k
tmax
MOMENT OF INERTIA (Eq. 5-47)  1.046 ;
taver
1
I (bh3  bh31 + th31)  1162 in.4 (d) SHEAR FORCE IN THE WEB (EQ. 5-49)
12
th1
(a) MAXIMUM SHEAR STRESS IN THE WEB (Eq. 5-48a) Vweb  (2tmax + tmin)  20.19 k ;
3
V
tmax  (bh2  bh21 + th21)  2634 psi ; Vweb
8It  0.961 ;
V
05Ch05.qxd 9/25/08 2:29 PM Page 460

460 CHAPTER 5 Stresses in Beams (Basic Topics)

Problem 5.10-6 Dimensions of cross section: b  120 mm, t  7 mm,


h  350 mm, h1  330 mm, and V  60 kN

Solution 5.10-6 Wide-flange beam


(b) MINIMUM SHEAR STRESS IN THE WEB (Eq. 5-48)
b  120 mm
Vb 2
t  7 mm tmin  (h  h21)  19.35 MPa ;
8It
h  350 mm (c) AVERAGE SHEAR STRESS IN THE WEB (Eq. 5-50)
h1  330 mm V
taver   25.97 MPa ;
V  60 kN th1
tmax
 1.093 ;
MOMENT OF INERTIA (Eq. 5-47) taver
1 (d) SHEAR FORCE IN THE WEB (Eq. 5-49)
I (bh3 bh31 + th31)  90.34 * 106 mm4
12 th1
Vweb  (2tmax + tmin)  58.63 kN ;
(a) MAXIMUM SHEAR STRESS IN THE WEB (Eq. 5-48a) 3
V Vweb
tmax  (bh2  bh21 + th21)  28.40 MPa  0.977 ;
8It V
;

Problem 5.10-7 A cantilever beam AB of length L  6.5 ft supports a q q


trapezoidal distributed load of peak intensity q, and minimum intensity —
2
q/2, that includes the weight of the beam (see figure). The beam is a
steel W 12  14 wide-flange shape (see Table E-1(a), Appendix E). B W 12  14
Calculate the maximum permissible load q based upon A
(a) an allowable bending stress sallow  18 ksi and (b) an allowable L = 6.5 ft
shear stress tallow  7.5 ksi. (Note: Obtain the moment of inertia
and section modulus of the beam from Table E-1(a))

Solution 5.10-7
b  3.97 in. I  88.6 # in.4
a
q
+ qb L
2 3
t  0.2 in. Vmax  Vmax  qL
2 4
t f  0.225 in.
1q 2 1 q 2L
Mmax  L + L
S  14.9 in.3 22 22 3
h  11.9 in. 5
Mmax  qL2
h1  h  2 tf 12

h1  11.45 in.
L  6.5 ft s allow  18 ksi t allow  7.5 ksi
05Ch05.qxd 9/25/08 2:29 PM Page 461

SECTION 5.10 Shear Stresses in Beams with Flanges 461

1 bh2  bh21 + th212


(a) MAXIMUM LOAD BASED UPON BENDING STRESS 3 qL

5 2 32It
qL
M 12 12S sallow tallow32It
s  q q
S S 5L2 3 L1 bh2  bh21 + th212
q  1270 lb/ft lb
q  3210
(b) MAXIMUM LOAD UPON SHEAR STRESS ft
q  1270 lb/ft
1 bh2  bh21 + th212
Vmax Shear stress governs ;
tmax 
8It

Problem 5.10-8 A bridge girder AB on a simple span of q


450 mm
length L  14 m supports a distributed load of maximum q q
— —
intensity q at midspan and minimum intensity q/2 at supports 2 2
A and B that includes the weight of the girder (see figure). 32 mm
A B
The girder is constructed of three plates welded to form the
cross section shown.
Determine the maximum permissible load q based upon L = 14 m
16 mm
(a) an allowable bending stress sallow  110 MPa and
1800 mm
(b) an allowable shear stress tallow  50 MPa.

32 mm

450 mm

Solution 5.10-8
L  14 m (a) MAXIMUM LOAD BASED UPON BENDING STRESS
h  1864 mm h1  1800 mm sallow  110 MPa
b  450 mm tf  32 mm tw  16 mm 3 L qLL qLL
Mmax  qL  
8 2 22 4 24 6
1 bh3  bh31 + tw h312
1
I
12 5
 qL2
I  3.194 * 10 10 4
mm 48
2I 5
S S  3.427 * 107 mm3 qL2
Mmax 48
h s 
S S
qL qL 3
RA  RB  +  qL sallow S
22 42 8 qmax  ;
5 2
L
48
kN
qmax  184.7 ;
m
05Ch05.qxd 9/25/08 2:29 PM Page 462

462 CHAPTER 5 Stresses in Beams (Basic Topics)

1 bh2  bh21 + th212


(b) MAXIMUM LOAD BASED UPON SHEAR STRESS 3 qL

tallow  50 MPa 64It
3 64 tallow Itw
qmax 
3 L 1bh2 bh12 tw h122
Vmax  R A  qL
8
qmax  247 kN/m
1 bh2  bh21 + th212
Vmax ;
tmax 
8It
‹ Bending stress governs: qmax 184.7 kN/m ;

Problem 5.10-9 A simple beam with an overhang supports a uniform P = 3000 lb


load of intensity q  1200 lb/ft and a concentrated load P  3000 lb P = 3000 lb
8 ft
(see figure). The uniform load includes an allowance for the wight q = 1200 lb/ft
of the beam. The allowable stresses in bending and shear are 18 ksi
and 11 ksi, respectively.
Select from Table E-2 (a), Appendix E, the lightest I-beam (S shape) A B
C
that will support the given loads.
(Hint: Select a beam based upon the bending stress and then
12 ft 4 ft
calculate the maximum shear stress. If the beam is overstressed in
shear, select a heavier beam and repeat.)

Solution 5.10-9 Beam with an overhand


sallow  18 ksi t allow  11 ksi L  12 ft Find moment at D (at Load P between A and B)
lb (8 ft)2
q  1200 P  3000 lb MD  R A 8 ft  q
ft 2
MD  1.28 * 104 lb-ft
Sum moments about A & Solve for RB
Mmax  | MB| Mmax  2.16 * 104 lb-ft
2
4 1
qa Lb + P(8 ft + 16 ft) Required section modulus:
3 2
RB  Mmax
12 ft S S  14.4 in.3
sallow
RB  1.88 * 104 lb
Lightest beam is S 8 * 23 (from Table E-2(a))
Sum forces in vertical direction I  64.7 in.4 S  16.2 in.3
RA  q (16 ft) + 2P  R B b  4.17 in. t  0.441 in.
R A  6.4 * 103 lb
t f  0.425 in. h  8 in.
Vmax  R B  (P + q4 ft)
h1  h  2 tf h1  7.15 in.
Vmax  1.1 * 104 lb at B
Check max. shear stress
(4 ft)2
MB   P (4 ft)  q
1 bh2  bh21 + th212
2 Vmax
tmax 
MB  2.16 * 10 lb-ft 4 8 It
tmax  3674 6 11,000 psi so ok for shear
Select S 8 * 23 beam ;
05Ch05.qxd 9/25/08 2:29 PM Page 463

SECTION 5.10 Shear Stresses in Beams with Flanges 463

Problem 5.10-10 A hollow steel box beam has the rectangular 20


cross section shown in the figure. Determine the maximum allowable mm
shear force V that may act on the beam if the allowable shear
stress in 36 Mpa.

10 mm 450
10 mm mm

20
mm

200 mm

Solution 5.10-10 Rectangular box beam


tallow  36 MPa
ba b  (180)a ba b
450 450 410 410
Q  (200)a
Find Vallow 2 4 2 4
VQ  1.280 * 106 mm3
t
It tallow It
Vallow 
tallowIt Q
Vallow 
Q (36 MPa)(484.9 * 106 mm4)(20 mm)

1 1 1.280 * 106 mm3
I (200)(450)3  (180)(410)3
12 12
 273 kN ;
 484.9*106mm4
t  2(10 mm)  20 mm

Problem 5.10-11 A hollow aluminum box beam has the square cross section 1.0 in.
shown in the figure. Calculate the maximum and minimum shear stresses tmax
and tmin in the webs of the beam due to a shear force V  28 k.
1.0 in.

12 in.
05Ch05.qxd 9/25/08 2:29 PM Page 464

464 CHAPTER 5 Stresses in Beams (Basic Topics)

Solution 5.10-11 Square box beam


b2 b b21 b1
Q a ba b  a ba b
2 4 2 4
1
V  28 k  28,000 lb  (b3  b31)  91.0 in.3
8
t1  1.0 in . VQ (28,000 lb)(91.0 in.3)
tmax    1424 psi
b  12 in. It (894.67 in.4)(2.0 in.)
b1  10 in.  1.42 ksi ;
MINIMUM SHEAR STRESS IN THE WEB (AT LEVEL A.A)
VQ
t t  2t1  2.0 in .
Q  Ay  (bt1)a  b  a b(b t1)
b t1 bt1
It
2 2 2
MOMENT OF INERTIA
b  b1 b
1 4 t1  Q  (b2  b21)
I (b  b41)  894.67 in.4 2 8
12 (12 in.)
Q [(12 in.)2  (10 in.)2]  66.0 in.3
MAXIMUM SHEAR STRESS IN THE WEB (AT NEUTRAL AXIS) 8
b2
Q  A1y1 A2y2 A1  ba b 
b VQ (28,000 lb)(66.0 in.3)
tmin    1033 psi
2 2 It (894.67 in4)(2.0 in.)
b21
A2  b1 a b
b1  1.03 ksi ;
2 2

y1  a b  y2  a b 
1 b b 1 b1 b1
2 2 4 2 2 4

Problem 5.10-12 The T-beam shown in the figure has cross-sectional dimensions y
as follows: b  220 mm, t  15 mm, h  300 mm, and h1  275 mm . The beam
is subjected to a shear force V  60 kN. t
Determine the maximum shear stress tmax in the web of the beam. h1 h
z C
c

b
Probs 5.10.12 and 5.-10.13

Solution 5.10-12
h  300 mm h1  280 mm LOCATION OF NEUTRAL AXIS
b  210 mm t  16 mm h  h1
b1 h  h12 a b + t h1 a h  b
h1
t f  h  h1 V  68 kN 2 2
c
t f  20 mm b1 h  h12 + t h1
c  87.419 mm
c1  c c1  87.419 mm
c2  h  c c2  212.581 mm
05Ch05.qxd 9/25/08 2:29 PM Page 465

SECTION 5.10 Shear Stresses in Beams with Flanges 465

MOMENT OF INERTIA ABOUT THE z-AXIS Iflange  2.531 * 107 mm4

t c + t1 c1  tf 23 I  Iweb + Iflange I  7.818 * 107 mm4


1 3 1
Iweb 
3 2 3 FIRST MOMENT OF AREA ABOVE THE z AXIS
Iweb  5.287 * 107 mm4 c2
Q  tc2
tf 2 2
b tf3 + b tf ac1  b
1
Iflange  VQ
12 2 tmax  tmax  19.7 MPa ;
It

Problem 5.10-13 Calculate the maximum shear stress tmax in the web
of the T-beam shown in the figure if b  10 in., t  0.5 in., h  7 in.,
h1  6.2 in., and the shear force V  5300 lb.

Solution 5.10-13 T-beam


h  7 in. h1  6.2 in. MOMENT OF INERTIA ABOUT THE z-AXIS
b  10 in. t  0.5 in.
t c 3 + t1 c1  tf23
1 1
Iweb 
tf  h  h1 tf  0.8 in. 3 2 3

V  5300 lb Iweb  29.656 in.4


tf 2
btf3 + btf a c1  b
LOCATION OF NEUTRAL AXIS 1
Iflange 
h  h1 12 2
b 1 h  h 12 a b + t h1 a h  b
h1
2 2 Iflange  8.07 in.4
c
b 1 h  h12 + t h1
I  Iweb + Iflange I  37.726 in.4
c  1.377 in.
FIRST MOMENT OF AREA ABOVE THE z AXIS
c1  c c1  1.377 in.
c2
c2  h  c c2  5.623 in. Q  tc2
2
VQ
tmax  tmax  2221 psi ;
It
05Ch05.qxd 9/25/08 2:29 PM Page 466

466 CHAPTER 5 Stresses in Beams (Basic Topics)

Built-Up Beams
Problem 5.11-1 A prefabricated wood I-beam serving as a floor joist y
has the cross section shown in the figure. The allowable load in shear
for the glued joints between the web and the flanges is 65 lb/in. in the 0.75 in.
longitudinal direction.
Determine the maximum allowable shear force Vmax for the beam.
z O 8 in.
0.625 in.

0.75 in.
5 in.

Solution 5.11-1 Wood I-beam


All dimensions in inches.
Find Vmax based upon shear in the glued joints.
Allowable load in shear for the glued joints is 65 lb/in.
‹ fallow  65 lb/in.
VQ fallow I
f Vmax  ;
I Q
bh3 (b  t)h31
I 
12 12
1 1
 (5) (9.5)3  (4.375)(8)3  170.57 in.4
12 12
Q  Qflange  Af df

 (5)(0.75)(4.375)  16.406 in.3

fallowI
Vmax 
Q

(65 lb/in.)(170.57 in.4)


  676 lb ;
16.406 in.3
05Ch05.qxd 9/25/08 2:29 PM Page 467

SECTION 5.11 Built-Up Beams 467

Problem 5.11-2 A welded steel girder having the cross section shown in the figure y
is fabricated of two 300 mm * 25 mm flange plates and a 800 mm * 16 mm
web plate. The plates are joined by four fillet welds that run continuously for the 25 mm
length of the girder. Each weld has an allowable load in shear of 920 kN/m.
Calculate the maximum allowable shear force Vmax for the girder.

z 800 mm
O
16 mm

25 mm
300 mm

Solution 5.11-2
h  850 mm h1  800 mm Qflange  3.094 * 106 mm3
b  300 mm t  16 mm kN
f allow  920 f  2 fallow
tf  25 mm m
b h3 (b  t)h13 (2 welds, one either side of web)
I 
12 12 VQ fI
f Vmax 
I  3.236 * 10 mm9 4
I Qflange
h  tf Vmax  1.924 MN ;
Qflange  A f df Qflange  b t f a b
2

y
Problem 5.11-3 A welded steel girder having the cross section shown in the figure
is fabricated of two 20 in. * 1 in. flange plates and a 60 in. * 5/16 in. web plate. 1 in.
The plates are joined by four longitudinal fillet welds that run continuously throughout
the length of the girder.
If the girder is subjected to a shear force of 280 kips, what force F (per inch of
length of weld) must be resisted by each weld?
z O 60 in.
5
— in.
16

1 in.
20 in.
05Ch05.qxd 9/25/08 2:29 PM Page 468

468 CHAPTER 5 Stresses in Beams (Basic Topics)

Solution 5.11-3
h  62 in. h1  60 in. h  tf
Qflange  btf a b
5 2
b  20 in. t in.
16 Qflange  610 in3
tf  1 in. VQ
V  280 k f  2F 
bh3 (b  t)h13 I
I 
12 12 VQflange
F F  1994 * 103 lb.in.
I  4.284 * 10 in.
4 4
21
Qflange  Af df F  1994 lb/in. ;

Problem 5.11-4 A box beam of wood is constructed of two y


260 mm * 50 mm boards and two 260 mm * 25 mm boards (see figure). 25 mm
The boards are nailed at a longitudinal spacing s  100 mm.
If each nail has a allowable shear force F  1200 N, what is the
maximum allowable shear force Vmax?
z 260 mm
O
50 50
mm mm

25 mm
260 mm

Solution 5.11-4 Wood box beam


All dimensions in millimeters. VQ fallow I
f Vmax 
b  260 b1  260  2(50)  160 I Q
h  310 h1  260 I
1
(bh3  b1h31)  411.125 * 106 mm4
s  nail spacing  100 mm 12
F  allowable shear force for one nail  1200 N Q  Qflange  Afdf  (260)(25)(142.5)
f  shear flow between one
flange and both webs  926.25 * 103 mm4

2F 2(1200 N) fallowI (24 kN/ m)(411.25 * 106 mm4)


fallow    24 kN/ m Vmax   .
s 100 mm Q 926.25 * 103 mm3
 10.7 kN ;
05Ch05.qxd 9/25/08 2:29 PM Page 469

SECTION 5.11 Built-Up Beams 469

Problem 5.11-5 A box beam is constructed of four wood y Flange Flange


boards as shown in the figure part (a). The webs are 8 in.  1 in.
and the flanges are 6 in.  1 in. boards (actual dimensions), 1 in. Web 1 in.
joined by screws for which the allowable load in shear is
F  250 lb per screw. Web
z O 8 in. 8 in.
(a) Calculate the maximum permissible longitudinal
spacing smax of the screws if the shear 1 in. 1 in.
force V is 1200 lb. 1 in. 1 in.
(b) Repeat (a) if the flanges are attached to the webs 6 in. 6 in.
using a horizontal arrangement of screws as shown 1 in. 1 in.
in the figure part (b). (a)
(b)

Solution 5.11-5 Wood box beam


V  1200 lb F  250 lb (b) Horizontal screws
(a) Vertical screws h  8 in. h1  6 in.
h  10 in. h1  8 in. b  8 in. t  1 in.

b  6 in. t  1 in. bh (b  2t) h13


3
I  I  233.333 in.4
(b  2t) h13
3 12 12
bh
I  I  329.333 in.4 Qb  (b  2 t) t (3.5 in.) Qb  21 in.3
12 12
Qa  bt (4.5 in.) Qa  27 in.3 VQ 2F
f 
VQ I s
2F
f  2FI
I S smax 
VQb
2FI
smax  smax  4.63 in. ;
VQa
smax  5.08 in. ;

Problem 5.11-6 Two wood box beams (beams A and B) y y


have the same outside dimensions (200 mm * 360 mm)
and the same thickness (t  20 mm) throughout, as shown
|in the figure on the next page. Both beams are formed by
nailing, with each nail having an allowable shear load of
A B
250 N. The beams are designed for a shear force V  3.2 kN.
z 360 z 360
(a) What is the maximum longitudinal spacing SA for the O mm O mm
nails in beam A? t= t=
(b) What is the maximum longitudinal spacing sB for the 20 mm 20 mm
nails in beam B?
(c) Which beam is more efficient in resisting the shear force? 200 mm 200 mm
05Ch05.qxd 9/25/08 2:29 PM Page 470

470 CHAPTER 5 Stresses in Beams (Basic Topics)

Solution 5.11-6 Two wood box beams


Cross-sectional dimensions are the same. (a) BEAM A
ht
b  (200)(20)a b(340)
All dimensions in millimeters. 1
Q  Af df  (bt)a
b  200 b1  200  2(20)  160 2 2
h  360 h1  360  2(20)  320  680 * 103 mm3
t  20 2FI (2)(250 N)(340.7 * 106 mm4)
sA  
F  allowable load per nail  250 N
VQ (3.2 kN)(680 * 103 mm3)
V  shear force  3.2 kN  78.3 mm ;
1 (b) BEAM B
I (bh3  b1 h31)  340.69 * 106 mm4
12 ht
Q  Afdf  (b  2t)(t)a b
s  longitudinal spacing of the nails 2
f  shear flow between one flange and both webs 1
 (160)(20) (340)
2F VQ 2FI 2
f  ‹ smax 
s I VQ  544 * 103 mm3
2FI (2)(250 N)(340.7 * 106 mm4)
sB  
VQ (3.2 kN)(544 * 103 mm3)
 97.9 mm ;
(c) BEAM B IS MORE EFFICIENT because the shear flow on
the contact surfaces is smaller and therefore fewer
nails are needed. ;

3 3
Problem 5.11-7 A hollow wood beam with plywood webs has the — in. — in.
16 16
cross-sectional dimensions shown in the figure. The plywood is attached
3 in.
to the flanges by means of small nails. Each nail has an allowable load
in shear of 30 lb.
Find the maximum allowable spacing s of the nails at cross sections
where the shear force V is equal to (a) 200 lb and (b) 300 lb. y
3
in.
4
z 8 in.
O
3
in.
4
05Ch05.qxd 9/25/08 2:29 PM Page 471

SECTION 5.11 Built-Up Beams 471

Solution 5.11-7 Wood beam with plywood webs


All dimensions in inches. (a) V  200 lb
b  3.375 b1  3.0 2FI 2(30 lb)(75.344 in.4)
smax  
h  8.0 h1  6.5 VQ (200 lb)(8.1563 in.3)
F  allowable shear force for one nail  30 lb  2.77 in. ;
s  longitudinal spacing of the nails (b) V  300 lb
f  shear flow between one flange and both webs By proportion,

smax  (2.77 in.) a b  1.85 in.


VQ 2F 2FI 200
f  ‹ smax  ;
I s VQ 300
1
I (bh3  b1h31)  75.3438 in.4
12
Q  Qflange  Afdf  (3.0)(0.75)(3.625)  8.1563 in.3

y
Problem 5.11-8 A beam of T cross section is formed by nailing together two boards
having the dimensions shown in the figure. 240 mm
If the total shear force V acting on the cross section is 1500 N and each nail may
carry 760 N in shear, what is the maximum allowable nail spacing s? 60 mm
z C
200 mm
60 mm

Solution 5.11-8
V  1500 N F allow  760 N MOMENT OF INERTIA ABOUT THE NEUTRAL AXIS
h1  200 mm b  240 mm
tc + t1 h1 c223
1 3 1
I
t  60 mm h  260 mm 3 2 3
t 2
bt + bt a c1  b
A  bt + h1t A  2.64 * 104 mm2 1 3
+
LOCATION OF NEUTRAL AXIS (z AXIS) 12 2

btah1  b + th1
t h1 I  1.549 * 108 mm4
2 2 FIRST MOMENT OF AREA OF FLANGE
c2 
A
Q  bt a c1  b
t
c2  170.909 mm 2
c1  h  c2 Q  8.509 * 105 mm3
c1  89.091 mm MAXIMUM ALLOWABLE SPACING OF NAILS
VQ F
f 
I s
F allowI
smax  smax  92.3 mm ;
VQ
05Ch05.qxd 9/25/08 2:29 PM Page 472

472 CHAPTER 5 Stresses in Beams (Basic Topics)

Problem 5.11-9 The T-beam shown in the figure is fabricated by welding y


together two steel plates. If the allowable load for each weld is 1.8 k/in. in the
longitudinal direction, what is the maximum allowable shear force V?

0.6 in.
5.5 in.
z C

4.5 in. 0.5 in.

Solution 5.11-9 T-beam (welded)


k MOMENT OF INERTIA ABOUT THE NEUTRAL AXIS
F allow  1.8
in.
t c 3 + t1 1 c2  t223
1 1
I
h1  5.5 in. b  4.5 in. 3 1 1 3
t1  0.6 in. t2  0.5 in. t2 2
b t23 + bt2 ac2  b
1
+
h  6 in. 12 2

A  bt2 + h1t1 A  5.55 in.2 I  20.406 in.4


LOCATION OF NEUTRAL AXIS (z AXIS) FIRST MOMENT OF AREA OF FLANGE
bt2 + t1 h1 a + t2 b
t2 h1
Q  b t2 ac2  b
t2
2 2 Q  4.014 in.3
c2  2
A
MAXIMUM ALLOWABLE SHEAR FORCE
c2  2.034 in. c1  h  c 2
VQ
f 2F
c1  3.966 in. I
2 Fallow I
Vmax 
Q
Vmax  18.30 k ;

Problem 5.11-10 A steel beam is built up from a W 410 * 85 wide-flange beam y


180 mm  9 mm
and two 180 mm * 9 mm cover plates (see figure). The allowable cover plates
load in shear on each bolt is 9.8 kN.
What is the required bolt spacing s in the longitudinal direction if the shear
force V = 110kN (Note: Obtain the dimensions and moment of inertia of the
W shape from Table E-1(b).) W 410  85
z
O
05Ch05.qxd 9/25/08 2:29 PM Page 473

SECTION 5.11 Built-Up Beams 473

Solution 5.11-10
V  110 kN F allow  9.8 kN 9 mm 2
+ Acp ac  b
W 410 * 85 2
Aw  10800 mm2 hw  417 mm I  4.57 * 108 mm4
Iw  310 * 106 mm4 First moment of area of one flange
Acp  (180) (9) (2) mm2 for two plates b
9 mm
Q  180 mm (9 mm)a c 
h  hw + (9 mm) (2) 2

A  Aw + Acp A  1.404 * 104 mm2 Q  3.451 * 105 mm3


Maximum allowable spacing of nails
LOCATION OF NEUTRAL AXIS (z AXIS)
VQ 2F
h f 
c c  217.5 mm I s
2
Moment of inertia about the neutral axis 2 Fallow I
smax  smax  236 mm ;
3
VQ
180 mm (9 mm)
I  Iw + (2)
12

Problem 5.11-11 The three beams shown have approximately the same cross-sectional area. Beam 1 is a W 14  82 with
flange plates; Beam 2 consists of a web plate with four angles; and Beam 3 is constructed of 2 C shapes with flange plates.
(a) Which design has the largest moment capacity?
(b) Which has the largest shear capacity?
(c) Which is the most economical in bending?
(d) Which is the most economical in shear?
Assume allowable stress values are: sa  18 ksi and ta  11 ksi. The most economical beam is that having the largest capacity-
to-weight ratio. Neglect fabrication costs in answering (c) and (d) above. (Note: Obtain the dimensions and properties of all rolled
shapes from tables in Appendix E.)

8  0.52
4  0.375

Four angles C 15  50
1 14  0.675
66—
W 14  82 2

4  0.375
8  0.52
Beam 1 Beam 2 Beam 3

Solution 5.11-11 Built-up steel beam


Beam 1: properties and dimensions for W14 * 82 with h1  hw + 2t1 bf1  10.1 in.
flange plates
tf1  0.855 in. tw1  0.51 in.
AW  24 in.2 hw  14.3 in. Iw  88l in.4
AI  AW + 2b1t1 AI  32.32 in.2
b1  8 in. t1  0.52 in.
05Ch05.qxd 9/25/08 2:29 PM Page 474

474 CHAPTER 5 Stresses in Beams (Basic Topics)

b1 + t31
Q1  b1 t1 a  b + bf1 tf1 a  b
t1 2 h1 t1 hw tf1
2 + b1 t1 a + b 2
hw
I1  Iw +
12 2 2 2 2 2 2
I1  1.338 * 103 in4 2
a  tf1 b
hw
2
Beam 2: properties and dimensions for L6 * 6 * 1/2 + tw1 Q1  98.983 in.3
angles with web plate 2
b2 2
a b
Aa  5.77 in.2 ca  1.67 in. ha  6 in.

Q2  2 Aa a  ca b + t2
Ia  19.9 in.4 b2  14 in. h2 2
2 2
t2  0.675 in. h2  b2
Q2  78.046 in. 3
A2  4Aa + b2t2 A2  32.53 in.2
Q3  b3 t3 a  b + 2bf3 tf3 a  b
2 t2 b32 h3 t3 hc tf3
I2  4Ia + Aa a  ca b 4 +
b2
2 12 2 2 2 2
2
a  tf3 b
I2  889.627 in.4 hc
2
Beam 3: properties and dimensions for C15 * 50 with + 2tw3 Q3  79.826 in.3
2
flange plates
I1 tw1
Ac  14.7 in.2 hc  15 in. Ic  404 in.4  4.448 * 103 mm2
Q1
b3  4 in. t3  0.375 in. h3  hc + 2t3
I2 t2
bf3  3.72 in. tf3  0.65 in. tw3  0.716 in.  4.964 * 103 mm2
Q2
A3  2Ac + 2b3 t3 A3  32.4 in.2
I2 2tw3
b3t33 t3 2
2 + b3 t3 a + b 2
hc  1.14 * 104 mm2 largest value
I3  Ic 2 + Q3
12 2 2
I3  985.328 in.4 Itw
Case (3) with maximum has the largest shear
capacity ; Q
(a) Beam with largest moment capacity; largest section
modulus controls (c) MOST ECONOMICAL BEAM IN BENDING HAS LARGEST
Mmax  sallow S BENDING CAPACITY-TO-WEIGHT RATIO

2I1 S3 S2
S1  S1  174.449 in.3 largest value  3.862 in. 6  3.907 in. 6
h1 A3 A2
2I2 S1
S2  S2  127.09 in.3  5.398 in.
h2 A1
2I3 Case (1) is the most economical in bending. ;
S3  S3  125.121 in.3
h3 (d) MOST ECONOMICAL BEAM IN SHEAR HAS LARGEST
case (1) with maximum S has the largest moment SHEAR CAPACITY-TO-WEIGHT RATIO
capacity ; I1 tw1 I2 t2
 0.213 6  0.237
(b) BEAM WITH LARGEST SHEAR CAPACITY: LARGEST Itw/Q Q1 A1 Q2 A2
RATIO CONTROLS
I3 tw3
tallow I tw 6  0.273
Q3 A3
Vmax 
O Case (3) is the most economical in shear. ;
05Ch05.qxd 9/25/08 2:29 PM Page 475

SECTION 5.12 Beams with Axial Loads 475

Problem 5.11-12 Two W 310 * 74 steel wide-flange beams are bolted together to form
a built-up beam as shown in the figure. What is the maximum permissible bolt spacing s
if the shear force V  80 kN and the allowable load in shear on each bolt is F  13.5 kN
(Note: Obtain the dimensions and properties of the W shapes from Table E-1(b).) W 310  74

W 310  74

Solution 5.11-12
V  80 kN W 310 * 74 FIRST MOMENT OF AREA OF FLANGE
F allow  13.5 kN A w  9420 mm 2 hw
Q  Aw Q  1.46 * 106 mm3
hw  310 mm I w  163 * 10 mm
6 4 2

Location of neutral axis (z axis) MAXIMUM ALLOWABLE SPACING OF NAILS


c  hw c  310 mm VQ 2F
f 
I s
MOMENT OF INERTIA ABOUT THE NEUTRAL AXIS
2Fallow I
hw 2
I  c Iw + Aw a b d (2)
smax  smax  180 mm ;
VQ
2
I  7.786 * 108 mm4

Beams with Axial Loads


When solving the problems for Section 5.12, assume that the bending
moments are not affected by the presence of lateral deflections. P = 25 lb

Problem 5.12-1 While drilling a hole with a brace and bit, you exert
a downward force P  25 lb on the handle of the brace (see figure).
The diameter of the crank arm is d  7/16 in. and its lateral offset
is b  4-7/8 in.
Determine the maximum tensile and compressive stresses st and d= —
7
sc, respectively, in the crank. 16 in.
7
b = 4—
8 in.
05Ch05.qxd 9/25/08 2:29 PM Page 476

476 CHAPTER 5 Stresses in Beams (Basic Topics)

Solution 5.12-1 Brace and bit


P  25 lb (compression) MAXIMUM STRESSES
M  Pb  (25 lb)(4 7/8 in.) P M 25 lb 121.9 lb-in.
st   +  2
+
 121.9 lb-in. A S 0.1503 in. 0.008221 in.3
d  diameter  166 psi + 14,828 psi  14,660 psi ;

d  7/16 in. P M
sc     166 psi  14,828 psi
A S
pd2
A  0.1503 in.2  14,990 psi ;
4
3
pd
S  0.008221 in.3
32

Problem 5.12-2 An aluminum pole for a street light weights 4600 N W2 = 660 N
and supports an arm that weights 660 N (see figure). The center of gravity
of the arm is 1.2 m from the axis of the pole. A wind force of 300 N also acts
in the (y) direction at 9 m above the base. The outside diameter of the
pole (at its base) is 225 mm, and its thickness is 18 mm. 1.2 m
Determine the maximum tensile and compressive stresses st and sc,
respectively, in the pole (at its base) due to the weights and the wind force.
P1 = 300 N W1 = 4600 N

18 mm
9m
z
y
x
y 225 mm
x

Solution 5.12-2
W1  4600 N b  1.2 m Mx  W2 b + P1h
W2  660 N h9m Mx  3.492 * 103 N # m (Moment)

P1  300 N d1  225 mm t  18 mm MAXIMUM STRESS

st  a  b
Pz Mx d1
d2  d1  2 t +
A I 2
1d1  d222 1d1 4  d2 42
p 2 p
A I st  5.77 * 103 kPa
4 64
 5770 kPa ;
A  1.171 * 104 mm2 I  6.317 * 107 mm4
sc  a  b
Pz Mx d1
AT BASE OF POLE  ;
A I 2
Pz  W1 + W2 sc  6.668 * 103
Pz  5.26 * 10 N 3
(Axial force )  6668 kPa ;
V y  P1 V y  300 N (Shear force)
05Ch05.qxd 9/25/08 2:29 PM Page 477

SECTION 5.12 Beams with Axial Loads 477

Problem 5.12-3 A curved bar ABC having a circular axis (radius h


B
r  12 in.) is loaded by forces P  400 lb (see figure). The cross
section of the bar is rectangular with height h and thickness t. A C
If the allowable tensile stress in the bar is 12,000 psi and the P P
height h  1.25 in., what is the minimum required thickness tmin ? 45° 45°

Solution 5.12-3 Curved bar


TENSILE STRESS
P M P 3Pr(2  12)
st  +  +
A S ht th2

c1 + 3(2  12) d
P r
r  radius of curved bar 
ht h
e  r  r cos 45°
MINIMUM THICKNESS
b
1
 ra1 
c1 + 3(2  12) d
12 P r
tmin 
hsallow h
Pr
M  Pe  (2  12)
2 SUBSTITUE NUMERICAL VALUES:

CROSS SECTION P  400 lb s allow  12,000 psi

1 2 r  12 in. h  1.25 in.


h  height t  thickness A  ht S  th
6 tmin  0.477 in. ;

B
Problem 5.12-4 A rigid frame ABC is formed by welding two
steel pipes at B (see figure). Each pipe has cross-sectional area
d d
A  11.31 * 103 mm2, moment of inertia I  46.37 * 106 mm4, P
H
and outside diameter d  200 mm. A C
Find the maximum tensile and compressive stresses st and sc,
respectively, in the frame due to the load P  8.0 kN if L  H  1.4 m. d

L L
05Ch05.qxd 9/25/08 2:29 PM Page 478

478 CHAPTER 5 Stresses in Beams (Basic Topics)

Solution 5.12-4 Rigid frame


P
AXIAL FORCE: N  RA sin a  sin a
2
PL
BENDING MOMENT: M  RAL 
2
TENSILE STRESS
N Mc P sin a PLd
st   +  +
A I 2A 4I
Load P at midpoint B
SUBSTITUTE NUMERICAL VALUES
P
REACTIONS: RA  RC  P  8.0 kN L  H  1.4 m a  45°
2
sina  1/12 d  200 mm
BAR AB:
A  11.31 * 103 mm2 I  46.37 * 106 mm4
H
tan a  (8.0 kN)(1/12)
L st  
H 2(11.31 * 103 mm2)
sin a 
1H2 + L2 (8.0 kN)(1.4 m)(200 mm)
+
d  diameter 4(46.37 * 106 mm4)
c  d/2  0.250 MPa + 12.08 MPa
 11.83 MPa (tension) ;
N Mc
sc     0.250 MPa  12.08 MPa
A I
 12.33 MPa (compression) ;

Problem 5.12-5 A palm tree weighing 1000 lb is inclined


at an angle of 60° (see figure). The weight of the tree may be
resolved into two resultant forces, a force P1  900 lb acting at
a point 12 ft from the base and a force P2  100 lb acting at
the top of the tree, which is 30 ft long. The diameter at the base
of the tree is 14 in. 30 ft P2 = 100 lb
Calculate the maximum tensile and compressive stresses
st and sc, respectively, at the base of the tree due to its weight.

12 ft P1 = 900 lb

60°
05Ch05.qxd 9/25/08 2:29 PM Page 479

SECTION 5.12 Beams with Axial Loads 479

Solution 5.12-5 Palm tree


M  P1L1 cos 60° P2 L2 cos 60°
 [(900 lb)(144 in.) + (100 lb)(360 in.)] cos 60°
 82,800 lb-in.
N  (P1 + P2) sin 60°  (1000 lb) sin 60°  866 lb

FREE-BODY DIAGRAM MAXIMUM TENSILE STRESS


N M 866 lb 82,800 lb-in.
P1  900 lb st   +  +
2
A S 153.94 in. 269.39 in.3
P2  100 lb
 5.6 psi + 307.4 psi  302 psi ;
L1  12 ft  144 in.
MAXIMUM COMPRESSIVE STRESS
L2  30 ft  360 in.
sc  5.6 psi  307.4 psi  313 psi ;
d  14 in.
pd2
A  153.94 in.2
4
pd3
S  269.39 in.3
32

Problem 5.12-6 A vertical pole of aluminum is fixed at the base and pulled 1.5 d2
at the top by a cable having a tensile force T (see figure). The cable is attached
at the outer edge of a stiffened cover plate on top of the pole and makes an
angle a  20° at the point of attachment. The pole has length L  2.5 m and a
hollow circular cross section with outer diameter d2  280 mm and inner
diameter d1  220 mm. The circular cover plate has diameter 1.5d2. a T
Determine the allowable tensile force Tallow in the cable if the allowable
compressive stress in the aluminum pole is 90 MPa. L
d1
d2
d2

Solution 5.12-6
sallow  90 MPa d1  220 mm PN  T cos (a) (Axial force)
d2  280 mm V  T sin (a) (Shear force)
d2  d1
M  VL + PN a b
1.5 d2
t a  20° L  2.5 m (Moment).
2 2

1d 2  d1 22 1d 4  d1 42
p p
A I
4 2 64 2
A  2.356 * 104 mm2 I  1.867 * 108 mm4
05Ch05.qxd 9/25/08 2:29 PM Page 480

480 CHAPTER 5 Stresses in Beams (Basic Topics)

Allowable Tensile Force sallow


Tallow 
sin (a) L + cos (a) a b
PN M d2 T cos (a) 1.5 d2
sc     cos (a) 2 d2
A I 2 A
+
A I 2
T sin (a) LT cos (a) a b
1.5 d2
2 d2 Tallow  108.6 kN ;

I 2

Problem 5.12-7 Because of foundation settlement,


a circular tower is leaning at an angle a to the vertical
(see figure). The structural core of the tower is a circular
cylinder of height h, outer diameter d2, and inner diameter d1.
For simplicity in the analysis, assume that the weight
of the tower is uniformly distributed along the height.
Obtain a formula for the maximum permissible angle
a if there is to be no tensile stress in the tower. h d1
d2

Solution 5.12-7 Leaning tower


CROSS SECTION
p 2
A (d  d21)
4 2
p 4
I (d  d41)
W  weight of tower 64 2
a  angle of tilt p 2
 (d  d21)(d22 + d21)
64 2
I d22 + d21

A 16
d2
c
2
AT THE BASE OF THE TOWER

N  W cos a M  Wa bsin a
h
2
05Ch05.qxd 9/25/08 2:29 PM Page 481

SECTION 5.12 Beams with Axial Loads 481

TENSILE STRESS (EQUAL TO ZERO) cos a hd2 sin a 4I d22 + d12


‹  tan a  
N Mc Wcosa A 4I hd2A 4hd2
st   + 
A I A
MAXIMUM ANGLE a
d22 + d12
+ a sinab a b  0
W h d2
a  arctan ;
I 2 2 4hd2

Problem 5.12-8 A steel bar of solid circular cross section and length y
L  2.5 m is subjected to an axial tensile force T  24 kN and a
bending moment M  3.5 kN m (see figure). d
(a) Based upon an allowable stress in tension of 110 MPa, determine M
z –z-direction
the required diameter d of the bar; disregard the weight of
the bar itself.
(b) Repeat (a) including the weight of the bar. T x
L

Solution 5.12-8
M  3.5 kN # m T  24 kN SOLVE NUMERICALLY FOR d (SUBSTITUTE sallow)
kN d  70 mm ;
g steel  77 3
L  2.5 m
m (b) INCLUDE WEIGHT OF BAR
sallow  110 MPa Agsteel L2
Mmax  M +
p 2 d p 4 2
A d c I d
4 2 64 AT TOP OF BEAM AT SUPPORT
(a) DISREGARD WEIGHT OF BAR T Mmax d
st  sallow  +
MAX. TENSILE STRESS AT TOP OF BEAM AT SUPPORT A I 2
T Md T M d SUBSTITUTE MMAX FROM ABOVE, SOLVE FOR d NUMERICALLY
smax  +  +
A I 2 p 2 p 42 d  76.5 mm ;
d d
4 64
4T 32 M
sallow  2
+
pd p d3
05Ch05.qxd 9/25/08 2:29 PM Page 482

482 CHAPTER 5 Stresses in Beams (Basic Topics)

Problem 5.12-9 A cylindrical brick chimney of height H weighs


w  825 lb/ft of height (see figure). The inner and outer diameters are
d1  3 ft and d2  4 ft, respectively. The wind pressure against the p
side of the chimney is p = 10 lb/ft2 of projected area. w
Determine the maximum height H if there is to be no tension in H
the brickwork
d1
d2

Solution 5.12-9 Brick Chimney


p 4 p 2
d2 I (d2  d41)  (d  d21) (d22  d21)
64 64 2
I 1 2 d2
 (d2 + d21) c
H q A 16 2
w
AT BASE OF CHIMNEY

M  qH a b  pd2 H2
H 1
N  W  wH
2 2

V TENSILE STRESS (EQUAL TO ZERO)


M N Md2 M 2I
s1   + 0 or 
N A 2I N Ad2
pd2 H2 d22 + d12

p  wind pressure 2wH 8d2
q  intensity of load  pd2 w(d22 + d21)
SOLVE FOR H H ;
d2  outer diameter 4pd22
d1  inner diameter SUBSTITUTE NUMERICAL VALUES
W  total weight of chimney  wH w  825 lb/ft d2  4 ft d1  3 ft
CROSS SECTION q  10 lb/ft2
Hmax  32.2 ft ;
p 2
A (d2  d21)
4
05Ch05.qxd 9/25/08 2:29 PM Page 483

SECTION 5.12 Beams with Axial Loads 483

Problem 5.12-10 A flying buttress transmits a load P  25 kN, acting


at an angle of 60° to the horizontal, to the top of a vertical buttress AB Flying
(see figure). The vertical buttress has height h  5.0 m and rectangular buttress
cross section of thickness t  1.5 m and width b  1.0 m (perpendicular
to the plane of the figure). The stone used in the construction weighs
y  26 kN/m3. P
W
What is the required weight W of the pedestal and statue above the
vertical buttress (that is, above section A) to avoid any tensile stresses 60°
in the vertical buttress? A
A

—t
h 2 h
t t
B B

Solution 5.12-10 Flying buttress


FREE-BODY DIAGRAM OF VERTICAL BUTTRESS CROSS SECTION
A  bt  (1.0 m)(1.5 m)  1.5 m2
1 1
S  bt2  (1.0 m)(1.5 m)2  0.375 m3
6 6
AT THE BASE
N  W + WB + P sin 60°
 W + 195 kN + (25 kN) sin 60°
 W + 216.651 kN
M  (Pcos 60°) h  (25 kN) (cos 60°) (5.0 m)
 62.5 kN # m
TENSILE STRESS (EQUAL TO ZERO)
P  25 kN
N M
h  5.0 m st   +
A S
t  1.5 m W + 216.651 kN 62.5 kN # m
 + 0
b  width of buttress perpendicular to the figure 1.5 m 2
0.375 m3
b  1.0 m or W  216.651 kN + 250 kN  0
g  26 kN/m 3 W  33.3 kN ;
WB  weight of vertical buttress
 bthg
 195 kN
05Ch05.qxd 9/25/08 2:29 PM Page 484

484 CHAPTER 5 Stresses in Beams (Basic Topics)

Problem 5.12-11 A plain concrete wall (i.e., a wall with no steel t


reinforcement) rests on a secure foundation and serves as a small
dam on a creek (see figure). The height of the wall is h  6.0 ft
and the thickness of the wall is t  1.0 ft.
(a) Determine the maximum tensile and compressive stresses st and
sc, respectively, at the base of the wall when the water level reaches the
top (d  h). Assume plain concrete has weight density gc  145 Ib/ft3. h
(b) Determine the maximum permissible depth dmax of the water if
d
there is to be no tension in the concrete.

Solution 5.12-11 Concrete wall

h  height of wall STRESSES AT THE BASE OF THE WALL


(d  DEPTH OF WATER)
t  thickness of wall
W M d 3gw
b  width of wall (perpendicular to the figure) st   +  hgc + Eq. (1)
A S t2
gc  width density of concrete
W M d 3gw
gw  weight density of water sc     hgc  2 Eq.(2)
A S t
d  depth of water
W  weight of wall (a) STRESSES AT THE BASE WHEN d  h
W  bhtgc h  6.0 ft  72 in. d  72 in.
F  resultant force for the water pressure t  1.0 ft  12 in.
MAXIMUM WATER PRESSURE = gw d 145
gc  145 lb/ft3  lb/in.3
1 1 1728
F (d)(gw d) (b)  bd2gw 62.4
2 2 gw  62.4 Ib/ft3  lb/in.3
1728
M  Fa b  bd3gw
d 1
3 6
1
A  bt S  bt2
6
05Ch05.qxd 9/25/08 2:29 PM Page 485

SECTION 5.12 Beams with Axial Loads 485

Substitute numerical values into Eqs. (1) and (2):


d3  (72 in.)(12 in.)2 a b  24,092 in.3
145
st  6.042 psi + 93.600 psi  87.6 psi ; 62.4
sc  6.042 psi  93.600 psi  99.6 psi ; dmax  28.9 in. ;

(b) MAXIMUM DEPTH FOR NO TENSION


Set st = 0 in Eq. (1):
d3gw
d3  ht2 a b
gc
hgc + 2
0
t gw

Problem 5.12-12 A circular post, a rectangular post, and a post of cruciform


cross section are each compressed by loads that produce a resultant force P acting
at the edge of the cross section (see figure). The diameter of the circular post and the
depths of the rectangular and cruciform posts are the same.
(a) For what width b of the rectangular post will the maximum tensile
stresses be the same in the circular and rectangular posts?
(b) Repeat (a) for the post with cruciform cross section.
(c) Under the conditions described in parts (a) and (b), which post has the
largest compressive stress?

P P P

b
x 4 — = b
4

b Load P here
d
4 — = d
d d d 4

Solution 5.12-12
(a) EQUAL MAXIMUM TENSILE STRESSES P M
COMPRESSION sc   
CIRCULAR POST A S

p 2 p 3 Pd 4P 16 P 20P
A d S d M  2
 2

4 32 2 pd pd pd 2
Tension RECTANGULAR POST
P M 4P 16 P 12 P bd2 Pd
st   +  2 + 2
 A  bd S M
A S pd pd pd2 6 2
P M P 3P 2P
TENSION st   +   
A S bd bd bd
05Ch05.qxd 9/25/08 2:29 PM Page 486

486 CHAPTER 5 Stresses in Beams (Basic Topics)

P M P 3P 4P Equate compressive stresses & solve for b


COMPRESSION s c      
A S bd bd bd 12 P 2P 3 1 pd
2
  b ;
Equate tensile stress expressions, solve for b pd 3bd pd b 3
12 P 2P 6 1 pd (c) THE LARGEST COMPRESSIVE STRESS
2
  b ;
pd bd pd b 6 substitute expressions for b above & compare
(b) CRUCIFORM CROSS SECTION compressive stresses
CIRCULAR POST
A  cbd  a bd
bd
22 20 P
sc  
3 3 pd2
S c + a b d 
bd b d 1 2 3
bd 2
2 12 2 2 12 d 32 RECTANGULAR POST
Pd 16P 4P 24 P
M  sc   
2 a bd2 b
3bd pd 2
a bd
3 pd
32 6
P M CRUCIFORM POST
TENSION st   +
A S 20 P 20 P
sc   
4P 16 P 12 P pd pd 2
 +  3 d
3bd 3bd 3bd 3
P M Rectangular post has the largest compressive
COMPRESSION sc    stress ;
A S
4P 16 P 20 P
  
3bd 3bd 3bd

Problem 5.12-13 Two cables, each carrying a tensile force


P  1200 lb, are bolted to a block of steel (see figure). The b
P P
block has thickness t  1 in. and width b  3 in.
t
(a) If the diameter d of the cable is 0.25 in., what are the maximum tensile
and compressive stresses st and sc, respectively, in the block?
(b) If the diameter of the cable is increased (without changing
the force P), what happens to the maximum tensile and compressive stresses?

Solution 5.12-13 Steel block loaded by cables

P  1200 lb d  0.25 in. b  width of block


t d  3.0 in.
t  1.0 in. e  +  0.625 in.
2 2
05Ch05.qxd 9/25/08 2:29 PM Page 487

SECTION 5.12 Beams with Axial Loads 487

CROSS SECTION OF BLOCK MAXIMUM COMPRESSIVE STRESS (AT BOTTOM OF BLOCK)


1 3 t
A  bt  30 in.2 I bt  0.25 in.4 y    0.5 in.
12 2
P Pey
(a) MAMIMUM TENSILE STRESS (AT TOP OF BLOCK) sc  +
A I
t
y  0.5 in. 1200 lb (1200 lb)(0.625 in.)( 0.5 in.)
2  +
P Pey 3 in.2 0.25 in.4
st  +
A I  400 psi  1500 psi  1100 psi ;
1200 lb (1200 lb)(0.625 in.)(0.5 in.)
 + (b) IF d IS INCREASED, increase the eccentricity e
3 in.2 0.25 in.4 increases and both stresses in magnitude.
 400 psi + 1500 psi  1900 psi ;

Problem 5.12-14 A bar AB supports a load P acting at the centroid b



of the end cross section (see figure). In the middle region of the bar 2
the cross-sectional area is reduced by removing one-half of the bar.
(a) If the end cross sections of the bar are square with sides of A b
b
length b, what are the maximum tensile and compressive stresses st
and sc, respectively, at cross section mn within the reduced region? b
(b) If the end cross sections are circular with diameter b, what b
— (a)
are the maximum stresses st and sc? 2
m n b

2

P
b

(b)

Solution 5.12-14 Bar with reduced cross section


(a) SQUARE BAR (b) CIRCULAR BAR
Cross section mn is a rectangle. Cross section mn is a semicircle
b2 b 3 b4 1 pb2 pb2
A  (b)a b  (b)a b  A a b
b 1
I  0.3927 b2
2 2 12 2 96 2 4 8

M  Pa b
b b From Appendix D, Case 10:
c
4 4 b 4
I  0.1098a b  0.006860 b4
STRESSES 2

b  0.2122 Pb
P Mc 2P 6P 8P 2b
st  +  2 + 2  2 ; M  Pa
A I b b b 3p
P Mc 2P 6P 4P
sc    2  2  2 ;
A I b b b
05Ch05.qxd 9/25/08 2:29 PM Page 488

488 CHAPTER 5 Stresses in Beams (Basic Topics)

FOR TENSION P P P
 2.546 2
+ 6.564 2
 9.11 ;
4r 2b b b b2
ct    0.2122 b
3p 3p P Mc c
sc  
FOR COMPRESSION: A I
P (0.2122 Pb)(0.2878 b)
b 2b  
cc  r  ct    0.2878 b 0.3927 b 2
0.006860 b4
2 3p
P P P
STRESSES  2.546 2
8.903 2
 6.36 ;
b b b2
P Mct P (0.2122 Pb)(0.2122 b)
st  +  +
A I 0.3927 b2 0.006860 b4

Problem 5.12-15 A short column constructed P = 25 k y


of a W 12 * 35 wide-flange shape is subjected to a resultant
C 10  15.3
compressive load P  12 k having its line of action at the
z (Part c only)
midpoint of one flange (see figure).
(a) Determine the maximum tensile and compressive
stresses st and sc, respectively, in the column. C
(b) Locate the neutral axis under this loading condition. 2
(c) Recompute maximum tensile and compressive stresses
if a C 10  15.3 is attached to one flange, as shown.
W 12  35
1 1

Solution 5.12-15 Column of wide-flange shape


PROPERTIES OF EACH SHAPE: P Pew
sc    c sc  5711 psi ;
W 12 * 35 C 10 * 15.3 Aw Iw w
Aw  10.3 in.3 Ac  4.48 in.2
(b) NEUTRAL AXIS (W SHAPE ALONE)
hw  12.5 in. twc  0.24 in.
Iw
tf  0.52 in. xp  0.634 in. y0   y0  4.62 in. ;
Aw ew
Iw  285 in. 4
Ic  2.27 in. (2-2 axis)
4

(a) THE MAXIMUM TENSILE AND COMPRESSIVE STRESSES (C) COMBINED COLUMN, W 12 * 35 with C 10 * 15.3
LOCATION OF CENTROID FOR W 12  35 ALONE h  hw + twc
hw h  12.74 in.
cw  cw  6.25 in.
2 A  Aw + Ac A  14.78 in.2
hw tf
P  25 k ew   ew  5.99 in.
2 2
P Pew
st   + c st  857 psi ;
Aw Iw w
05Ch05.qxd 9/25/08 2:29 PM Page 489

SECTION 5.12 Beams with Axial Loads 489

LOCATION OF CENTROID OF COMBINED SHAPE tf


P  25 k e  hw  c e  4.215 in.
Aw a b + Ac (h  xp)
hw 2
2 P Pe
c c  8.025 in. st   + c st  453 psi ;
A A I
hw 2
I  Iw + Aw ac  b
P Pe
sc    (h  c) sc  2951 psi ;
2 A I
+ Ic + Ac (h  xp  c)2 I
y0   y0  6.33 in. (from centroid) ;
I  394.334 in.4 Ae

Problem 5.12-16 A short column of wide-flange shape is


P = 55 kN y Cover plate
subjected to a compressive load that produces a resultant force (120 mm  10 mm)
P  55 kN acting at the midpoint of one flange (see figure). (Part c only)
z y
(a) Determine the maximum tensile and compressive
stresses st and sc, respectively, in the column. P
(b) Locate the neutral axis under this loading condition. C
(c) Recompute maximum tensile and compressive stresses 8 mm
if a 120 mm  10 mm cover plate is added to one flange z C
200
mm
as shown.
12 mm
160
mm

Solution 5.12-16
P  55 kN d tf
e  e  94 mm
(a) MAXIMUM TENSILE AND COMPRESSIVE STRESSES FOR W 2 2
SHAPE ALONE P Pe d
st   + st  3.27 MPa ;
PROPERTIES AND DIMENSIONS FOR W SHAPE Aw Iw 2
b  160 mm d  200 mm P Pe d
sc    sc  24.2 MPa ;
tf  12 mm tw  8 mm Aw Iw 2

Aw  bd  (b  tw) (d  2 tf) (b) NEUTRAL AXIS (W SHAPE ALONE)


Aw  5.248 * 103 mm2 Iw
(b  tw) (d  2 tf)3 y0   y0  76.2 mm ;
bd3 Aw e
Iw  
12 12
Iw  3.761 * 107 mm4 (c) COMBINED COLUMN-W SHAPE & COVER PLATE
bp  120 mm tp  10 mm
05Ch05.qxd 9/25/08 2:29 PM Page 490

490 CHAPTER 5 Stresses in Beams (Basic Topics)

h  dtp I  4.839 * 107 mm4


h  210 mm tf
ed c e  74.459 mm
A  Aw + bp tp A  6.448 * 103 mm2 2
CENTROID OF COMPOSITE SECTION P Pe
st   + c st  1.587 MPa ;
tp A I
Aw + bp tp ad + b
d
2 2 P Pe
c sc    (h  c) sc  20.3 MPa ;
A Aw Iw
c  119.541 mm NEUTRAL AXIS
2
I  Iw + Aw ac  b
d I
y0   y0  100.8 mm (from centrioid)
2 Ae
bp t3p tp 2
+ + bp tp a d +  cb
12 2

Problem 5.12-17 A tension member constructed of an 2


1
L 4  4  2 inch angle section (see Table E-4(a) in 3 1 1
Appendix E) is subjected to a tensile load P  12.5 kips L44— 2L44—
2 2
that acts through the point where the midlines of the legs C C
intersect [see figure part (a)]. 1 1
P  
(a) Determine the maximum tensile stress st in the
angle section. 2 3 P
(b) Recompute the maximum tensile stress if two angels
are used and P is applied as shown in the figure part (b). (a) (b)

Solution 5.12-17 Angle section in tension


(a) ONE ANGLE: L 4 * 4 * 1/2 (b) TWO ANGLES: L 4 * 4 * 1/2
AL  3.75 in.2 rmin  0.776 in. A  2AL
t  0.5 in. t  0.5 in
c  1.18 in. c  1.18 in
IL  5.52 in.4 (2-2 axis)
e ac b 12
t
e  1.315 in
2
e ac b
t
e  0.93 in.
P  12.5 k 2
c1  c 12 c1  1.699 in. P  12.5 k
I3  AL rmin2 I3  2.258 in. 4 I  2IL I  11.04 in.4
M  Pe M  16.44 k-in. M  Pe M  11.625 k-in.
MAXIMUM TENSILE STRESS OCCURS AT CORNER MAXIMUM TENSILE STRESS OCCURS AT THE LOWER EDGE
P Mc1 P Mc
st  + st  15.48 ksi ; st  + st  2.91 ksi ;
AL I3 A I
05Ch05.qxd 9/25/08 2:29 PM Page 491

SECTION 5.12 Beams with Axial Loads 491

Problem 5.12-18 A short length of a 200 * 17.1 channel Two L 76  76  6.4 angles
is subjected to an axial compressive force P that has its line y y
of action through the midpoint of the web of the channel C 200 × 17.1
[(see figure(a)]. P P
 
 
(a) Determine the equation of the neutral axis under this z z
C C
loading condition.
(b) If the allowable stresses in tension and compression C 200 × 17.1
are 76 MPa and 52 MPa respectively, find the (a)
(b)
maximum permissible load Pmax.
(c) Repeat (a) and (b) if two L 76  76  6.4 angles are
added to the channel as shown in the figure part (b).
See Table E-3(b) in Appendix E for channel properties and Table E-4(b) for angle properties.

Solution 5.12-18
sc
P
1 e
tw = 5.59mm   c1
Ac Ic
bf = 57.4 Pmax  67.3 kN ;

C 200 * 17.1 (c) COMBINED COLUMN WITH 2-ANGLES

Ac  2170 mm2 dc  203 mm c1  14.5 mm L 76 * 76 * 6.4

Ic  0.545 * 106 mm4 (z-axis) AL  929 mm2 IL  0.512 * 106 mm4

c2  bf  c1 c2  42.9 mm cL  21.2 mm

ALLOWABLE STRESSES COMPOSITE SECTION


st  76 MPa s c  52 MPa A  Ac + 2 AL A  4.028 * 103 mm2
ECCENTRICITY OF THE LOAD h  bf + 76 mm h  133.4 mm
tw CENTROID OF COMPOSITE SECTION
e  c1  e  11.705 mm
2 Ac 1bf  c12 + 2 AL 1bf + cL2
c
A
(a) LOCATION OF THE NEUTRAL AXIS (CHANNEL ALONE)
c  59.367 mm
Ic
y0  y0  21.5 mm ; I  Ic + Ac 1bf  c1  c22
Ac # e
+ 2 IL + 2 AL 1bf + cL  c22
(b) FIND PMAX I  2.845 * 106 mm4
P Pe st tw
st   + c P e  bf  c e  4.762 mm
A I 2 1 e 2
 + c2
Ac Ic bf  57.4 mm
P  165.025 kN
LOCATION OF THE NEUTRAL AXIS
P Pe
sc    c I
A I 1 y0   y0  148.3 mm ;
Ae
05Ch05.qxd 9/25/08 2:29 PM Page 492

492 CHAPTER 5 Stresses in Beams (Basic Topics)

y0  148.3 mm 7 h  133.4 mm ; sc
P Pmax  149.6 kN ;
Thus, this composite section has no tensile stress 1 e
 + c
A I
P Pe
sc   + c
A I

Stress Concentrations
The problems for Section 5.13 are to be solved considering the
stress-concentration factors. M M
Problem 5.13-1 The beams shown in the figure are subjected h d
to bending moments M  2100 lb-in. Each beam has a rectangular
cross section with height h  1.5 in. and width b  0.375 in.
(perpendicular to the plane of the figure). (a)
(a) For the beam with a hole at midheight, determine the
maximum stresses for hole diameters d  0.25, 0.50, 0.75, 2R
and 1.00 in.
(b) For the beam with two identical notches (inside height M M
h1  1.25 in .), determine the maximum stresses for notch radii h h1
R  0.05, 0.10, 0.15, and 0.20 in.

Probs. 5.13.1 through 5.13-4 (b)

Solution 5.13-1
M  2100 lb-in. h  1.5 in. b  0.375 in. (b) BEAM WITH NOTCHES
(a) BEAM WITH A HOLE h 1.5 in.
h1  1.25 in.   1.2
h1 1.25 in.
d 1 6Mh
… Eq.(5-57): sc  Eq. (5-58)
h 2 b(h3  d3)
6M
50,400 snom   21,500 psi
 (1) bh21
3.375  d 3

d 1 12Md R sm ax  Ks nom
Ú Eq.(5-56): sB  K
h 2 b(h3  d3) R (in) h1 (Fig. 5-50) sm ax (psi)
67,200 d
 (2) 0.05 0.04 3.0 65,000
3.375  d3 0.10 0.08 2.3 49,000
0.15 0.12 2.1 45,000
0.20 0.16 1.9 41,000
d sc Eq. (1) sB sm ax
d (in.) h (psi) Eq. (2) (psi) (psi) NOTE: The larger the notch radius, the smaller the stress.
0.25 0.1667 15,000 — 15,000
0.50 0.3333 15,500 — 15,500
0.75 0.5000 17,100 17,100 17,100
1.00 0.6667 — 28,300 28,300
NOTE: The larger the hole, the larger the stress.
05Ch05.qxd 9/25/08 2:29 PM Page 493

SECTION 5.13 Stress Concentrations 493

Problem 5.13-2 The beams shown in the figure are subjected to bending
moments M  250 N # m. Each beam has a rectangular cross section with height
h  44 mm and width b  10 mm (perpendicular to the plane of the figure).
(a) For the beam with a hole at midheight, determine the maximum stresses
for hole diameters d  10, 16, 22 and 28 mm.
(b) For the beam with two identical notches (inside height h1  40 mm ),
determine the maximum stresses for notch radii R  2, 4, 6, and 8 mm.

Solution 5.13-2
M  250 N # m h  44 mm b  10 mm (b) BEAM WITH NOTCHES
(a) BEAM WITH A HOLE h 44 mm
h1  40 mm   1.1
d 1 h1 40 mm
… Eq. (5-57): 6M
h 2 Eq. (5-58): snom   93.8 MPa
6Mh 6.6 * 10 6 bh21
sc   MPa (1)
b(h3  d3) 85,180  d3
d 1 R R K smax  Ks nom
Ú Eq. (5-56): (mm) h1 (Fig. 5-50) smax (MPa)
h 2
12Md 300 * 103d 2 0.05 2.6 240
sB   MPa (2) 4 0.10 2.1 200
b(h3  d3) 85,180  d 3
6 0.15 1.8 170
8 0.20 1.7 160
sB NOTE: The larger the notch radius, the smaller the stress.
d sc Eq. (2) sm ax
d (mm) h Eq. (1) (MPa) (MPa) (MPa)
10 0.227 78 — 78
16 0.364 81 — 81
22 0.500 89 89 89
28 0.636 — 133 133
NOTE: The larger the hole, the larger the stress.

Problem 5.13-3 A rectangular beam with semicircular notches, as shown


in part (b) of the figure, has dimensions h  0.88 in. and h1  0.80 in. The
maximum allowable bending stress in the metal beam is smax  60 ksi, and
the bending moment is M  600 lb-in.
Determine the minimum permissible width bmin of the beam.
05Ch05.qxd 9/25/08 2:29 PM Page 494

494 CHAPTER 5 Stresses in Beams (Basic Topics)

Solution 5.13-3 Beam with semicircular notches


h  0.88 in. h1  0.80 in.
b
6M
smax  Ksnom  Ka
smax  60 ksi M  600 lb-in. bh21

d
1 6(600 lb-in.)
h  h1 + 2R R  (h  h1)  0.04 in. 60 ksi  2.57c
2 b(0.80 in.)2
R 0.04 in. Solve for b:
  0.05
h1 0.80 in. bmin L 0.24 in. ;
From Fig. 5-50: K L 2.57

Problem 5.13-4 A rectangular beam with semicircular notches,


as shown in part (b) of the figure, has dimension h  120 mm and
h1  100 mm . The maximum allowable bending stress in the plastic
beam is smax  6 MPa, and the bending moment is M  150 N # m.
Determine the minimum permissible width bmin of the beam.

Solution 5.13-4 Beam with semicircular notches


h  120 mm h1  100 mm
smax  Ksnom  K a b
6M
smax  6 MPa M  150 N # m bh21
1 6(150 N # m)
h  h1 + 2R R  (h  h1)  10 mm 6 MPa  2.20 c d
2 b(100 mm)2
R 10 mm Solve for b:
  0.10
h1 100 mm
bmin L 33 mm ;
From Fig.5-50: K L 2.20

Problem 5.13-5 A rectangular beam with notches and a 2R


hole (see figure) has dimensions h  5.5 in., h1  5 in., and width
b  1.6 in. The beam is subjected to a bending moment M
M
M  130 k-in., and the maximum allowable bending stress h1 h d
in the material (steel) is smax  42,000 psi.
(a) What is the smallest radius Rmin that should be
used in the notches?
(b) What is the diameter dmax of the largest hole that should
be drilled at the midheight of the beam?
05Ch05.qxd 9/25/08 2:29 PM Page 495

SECTION 5.13 Stress Concentrations 495

Solution 5.13-5 Beam with notches and a hole


h  5.5 in. h1  5 in. b  1.6 in. (b) LARGEST HOLE DIAMETER
M  130 k-in. smax  42,000 psi d 1
Assume 7 and use Eq. (5-56).
h 2
(a) MINIMUM NOTCH RADIUS
12Md
h 5.5 in. sB 
  1.1 b(h3  d3)
h1 5 in.
12(130 k-in.)d
6M 42,000 psi  or
snom   19,500 psi (1.6 in.)[(5.5 in.)3  d3]
bh21 d3 + 23.21d  166.4  0
smax 42,000 psi
K   2.15 Solve numerically:
snom 19,500 psi
dmax  4.13 in. ;
h
From Fig. 5-50, with K  2.15 and  1.1, we get
h1
R
L 0.090
h1
‹ Rmin L 0.090h1  0.45 in. ;
05Ch05.qxd 9/25/08 2:29 PM Page 496

You might also like